Med Surg FINAL

Pataasin ang iyong marka sa homework at exams ngayon gamit ang Quizwiz!

Which finding should the nurse expect when assessing a patient who has osteoarthritis (OA) of the knee? a. Presence of Heberden's nodules b. Discomfort with joint movement c. Redness and swelling of the knee joint d. Stiffness that increases with movement

B

Which information about a patient who had a stapedotomy yesterday is most important for the nurse to communicate to the health care provider? a. The patient reports ear "fullness." b. Oral temperature is 100.8° F (38.1° C). c. Small amount of dried drainage on dressing. d. The patient reports that hearing has gotten worse.

B

Which information should the nurse include in discharge teaching for a patient who has had a repair of a fractured mandible? a. Administration of nasogastric tube feedings b. How and when to cut the immobilizing wires c. The importance of high-fiber foods in the diet d. The use of sterile technique for dressing changes

B

A 25-yr-old female patient with systemic lupus erythematosus (SLE) has a facial rash and alopecia. She tells the nurse, "I never leave my house because I hate the way I look." Which GRADESMORE.COM patient problem should the nurse plan to address? a. Social isolation b. Activity intolerance c. Impaired skin integrity d. Impaired social interaction

A

A patient with atopic dermatitis has been using a high-potency topical corticosteroid ointment for several weeks. The nurse should assess for which adverse effect? a. Thinning of the affected skin b. Alopecia of the affected area c. Dryness and scaling in the area d. Reddish-brown skin discoloration

A

A patient with dermatomyositis is receiving long-term prednisone therapy. Which assessment finding should the nurse report immediately to the health care provider? a. The patient has painful hematuria. b. Acne is noted on the patient's face. c. Fasting blood glucose is 112 mg/dL. d. The patient has an increased appetite.

A

Ten days after receiving a bone marrow transplant, a patient develops a skin rash. What should the nurse suspect is the cause of the rash? a. The donor T cells are attacking the patient's skin cells. b. The patient needs treatment to prevent hyperacute rejection. c. The patient's antibodies are rejecting the donor bone marrow. d. The patient is experiencing a delayed hypersensitivity reaction.

A

The most common cause of secondary immunodeficiencies is a. drugs. b. stress. c. malnutrition. d. human immunodeficiency virus.

A

What should the occupational health nurse advise a patient whose job involves many hours of typing? a. Obtain a keyboard pad to support the wrist. b. Do stretching exercises before starting work. c. Wrap the wrists with compression bandages every morning. d. Avoid using nonsteroidal antiinflammatory drugs (NSAIDS).

A

Which safe sun practices would the nurse include in the teaching plan for a patient who has photosensitivity (select all that apply)? a. Wear protective clothing. b. Apply sunscreen liberally and often. c. Emphasize the short-term use of a tanning booth. d. Avoid exposure to the sun, especially during midday. e. Wear any sunscreen as long as it is bought at a drugstore.

A, B, D

During assessment of the patient with fibromyalgia, the nurse should expect the patient to report which of the following? (Select all that apply.) a. Sleep disturbances b. Multiple tender points c. Cardiac palpitations and dizziness d. Multijoint inflammation and swelling e. Widespread bilateral, burning musculoskeletal pain

A, B, E

Which activities can the nurse working in the outpatient clinic delegate to a licensed practical/vocational nurse (LPN/VN)? (Select all that apply.) a. Administer patch testing to a patient with allergic dermatitis. b. Interview a new patient about chronic health problems and allergies. c. Apply a sterile dressing after the health care provider excises a mole. d. Explain potassium hydroxide testing to a patient with a skin infection. e. Teach a patient about site care after a punch biopsy of an upper arm lesion.

A, C

While performing passive range of motion for a patient, the nurse puts the elbow joint through the movements of (select all that apply) a. flexion and extension. b. inversion and eversion. c. pronation and supination. d. flexion, extension, abduction, and adduction. e. pronation, supination, rotation, and circumduction.

A, C

Which actions should the nurse include in the plan of care for a patient with metastatic bone cancer of the left femur? (Select all that apply.) a. Monitor serum calcium. b. Teach about the need for strict bed rest. c. Explain the use of sustained-release opioids. d. Support the left leg when repositioning the patient. e. Assist family and patient as they discuss the prognosis.

A, C, D, E

Which laboratory result should the nurse monitor to determine if prednisone has been effective for a patient who has an acute exacerbation of rheumatoid arthritis? a. Blood glucose b. C-reactive protein c. Serum electrolytes d. Liver function tests

B

Type I: IgE mediated reactions are ...

Anaphylactic reactions

A 29-yr-old woman is taking methotrexate to treat rheumatoid arthritis. Considering this treatment, which information should the nurse report to the health care provider? a. The patient had a history of infectious mononucleosis as a teenager. b. The patient is trying to get pregnant before her disease becomes more severe. c. The patient has a family history of age-related macular degeneration of the retina. d. The patient has been using large doses of vitamins and health foods to treat the RA.

B

A 35-yr-old female patient states that she is using topical fluorouracil to treat actinic keratoses on her face. Which additional assessment information will be most important for the nurse to obtain? a. History of sun exposure by the patient b. Method of contraception used by the patient c. Length of time the patient has used fluorouracil d. Appearance of the treated areas on the patient's face

B

A patient who is to have no weight bearing on the left leg is learning to use crutches. Which observation by the nurse indicates the patient can safely ambulate independently? a. The patient moves the right crutch with the right leg and then the left crutch with the left leg. b. The patient advances the left leg and both crutches together and then advances the right leg. c. The patient uses the bedside chair to assist in balance as needed when ambulating in the room. d. The patient keeps the padded area of the crutch firmly in the axillary area when ambulating.

B

The nurse supervises the care of a patient with a temporary radioactive cervical implant. Which action by unlicensed assistive personnel (UAP), if observed by the nurse, would require an intervention? a. The UAP flushes the toilet once after emptying the patient's bedpan. b. The UAP stands by the patient's bed for 30 minutes talking with the patient. c. The UAP places the patient's bedding in the laundry container in the hallway. d. The UAP gives the patient an alcohol-containing mouthwash to use for oral care.

B

Which medication information should the nurse identify as a potential risk to a patient's musculoskeletal system? a. The patient takes a daily multivitamin and calcium supplement. b. The patient has asthma requiring frequent therapy with oral corticosteroids. c. The patient takes hormone replacement therapy (HRT) to prevent "hot flashes." d. The patient has headaches treated with nonsteroidal antiinflammatory drugs (NSAIDs).

B

Which lymphocyte is part of the Humoral immunity ?

B Lymphocytes

Age-related changes in the hair and nails include (select all that apply) a. oily scalp. b. scaly scalp. c. thinner nails. d. thicker, brittle nails. e. longitudinal nail ridging.

B, D, E

A common site for the lesions associated with atopic dermatitis is the a. buttocks. b. temporal area. c. antecubital space. d. plantar surface of the feet.

C

The nurse designs a program to decrease the incidence of human immunodeficiency virus (HIV) infection in the adolescent and young adult populations. Which information should the nurse assign as the highest priority? a. Methods to prevent perinatal HIV transmission. b. Ways to sterilize needles used by injectable drug users. c. Prevention of HIV transmission between sexual partners. d. Means to prevent transmission through blood transfusions.

C

The nurse is caring for a patient receiving intravesical bladder chemotherapy. The nurse should monitor for which adverse effect? a. Nausea b. Alopecia c. Hematuria d. Xerostomia

C

When the nurse is taking a health history of a new patient at the ear clinic, the patient states, "I have to sleep with the television on." Which follow-up question is appropriate to obtain more information about possible hearing problems? a. "Do you grind your teeth at night?" b. "What time do you usually fall asleep?" c. "Have you noticed ringing in your ears?" d. "Are you ever dizzy when you are lying down?"

C

Which nursing action will be most useful in assisting a young adult to adhere to a newly prescribed antiretroviral therapy (ART) regimen? a. Give the patient detailed information about possible medication side effects. b. Remind the patient of the importance of taking the medications as scheduled. c. Help the patient develop a schedule to decide when the drugs should be taken. d. Encourage the patient to join a support group for adults who are HIV positive.

C

Which patient who has arrived at the human immunodeficiency virus (HIV) clinic should the nurse assess first? a. Patient whose rapid HIV-antibody test is positive. b. Patient whose latest CD4+ count has dropped to 250/μL. c. Patient who has had 10 liquid stools in the last 24 hours. d. Patient who has nausea from prescribed antiretroviral drugs.

C

Which information should the nurse include when teaching a patient with acute low back pain? (Select all that apply.) a. Sleep in a prone position with the legs extended. b. Keep the knees straight when leaning forward to pick something up. c. Expect symptoms of acute low back pain to improve in a few weeks. d. Avoid activities that require twisting of the back or prolonged sitting. e. Use ibuprofen (Motrin, Advil) or acetaminophen (Tylenol) to relieve pain.

C, D, E

Type IV hypersensitivity is related to...

Cell mediated immunity

What are the three different types of cytokines?

Colony-Stimulating Factor Interferons Interleukins

A 54-yr-old woman who recently reached menopause and has a family history of osteoporosis is diagnosed with osteopenia. Which information should the nurse explain to the patient? a. With a family history of osteoporosis, there is no way to prevent or slow bone resorption. b. Estrogen replacement therapy must be started to prevent rapid progression to osteoporosis. c. Continuous, low-dose corticosteroid treatment is effective in stopping the course of osteoporosis. d. Calcium loss from bones can be slowed by increasing calcium intake and weight-bearing exercise.

D

A characteristic of the stage of progression in cancer development is a. oncogenic viral transformation of target cells. b. a reversible steady growth facilitated by carcinogens. c. a period of latency before clinical detection of cancer. d. proliferation of cancer cells despite host control mechanisms.

D

A patient has had surgical reduction of an open fracture of the right radius. Which assessment findings should the nurse report immediately to the health care provider? a. Serous wound drainage b. Right arm muscle spasms c. Pain with right arm movement D. Temperature 101.4 F (38.6 C)

D

A patient has possible right carpal tunnel syndrome. What symptom should the nurse expect with a positive Tinel's sign? a. Weakness in the right little finger b. Burning in the right elbow and forearm c. Tremor when gripping with the right hand d. Tingling in the right thumb and index finger

D

A patient is to be discharged from the hospital 4 days after insertion of a femoral head prosthesis using a posterior approach. Which patient statement to the nurse indicates that additional teaching is needed? a. "I should not cross my legs while sitting." b. "I will use a toilet elevator on the toilet seat." c. "I will have someone else put on my shoes and socks." d. "I can sleep in any position that is comfortable for me."

D

A patient receiving head and neck radiation for larynx cancer has ulcerations over the oral mucosa and tongue and thick, ropey saliva. Which instructions should the nurse give to this patient? a. Remove food debris from the teeth and oral mucosa with a stiff toothbrush. b. Use cotton-tipped applicators dipped in hydrogen peroxide to clean the teeth. c. Gargle and rinse the mouth several times a day with an antiseptic mouthwash. d. Rinse the mouth before and after each meal and at bedtime with a saline solution.

D

A patient reporting painful urination and knee pain is diagnosed with reactive arthritis. What long-term therapy should the nurse plan to explain to the patient? a. methotrexate b. anakinra (Kineret) c. etanercept (Enbrel) d. doxycycline (Vibramycin)

D

A patient undergoing external radiation has developed a dry desquamation of the skin in the treatment area. The nurse teaches the patient about the management of the skin reaction. Which statement, if made by the patient, indicates the teaching was effective? a. "I can use ice packs to relieve itching." b. "I will scrub the area with warm water." c. "I will expose my skin to a sun lamp each day." d. "I can buy some aloe vera gel to use on my skin."

D

An older adult patient with a squamous cell carcinoma (SCC) on the lower arm has a Mohs procedure in the dermatology clinic. Which nursing action will be included in the postoperative plan of care? a. Schedule daily appointments for dressing changes. b. Describe the use of topical fluorouracil on the incision. c. Instruct how to use sterile technique to clean the suture line. d. Teach the use of cold packs to reduce bruising and swelling.

D

The nurse is caring for a patient who smokes 2 packs/day. Which action by the nurse could help reduce the patient's risk of lung cancer? a. Teach the patient about the seven warning signs of cancer. b. Plan to monitor the patient's carcinoembryonic antigen (CEA) level. c. Teach the patient about annual chest x-rays for lung cancer screening. d. Discuss risks associated with cigarettes during each patient encounter.

D

The nurse reviews the laboratory results of a patient who is receiving chemotherapy. Which laboratory result is most important to report to the health care provider? a. Hematocrit 30% b. Platelets 95,000/μL c. Hemoglobin 10 g/L d. White blood cells (WBC) 2700/μL

D

The nurse is developing a health promotion plan for an older adult who worked in the landscaping business for 40 years. The nurse will plan to teach the patient about how to self-assess for which clinical manifestations? (Select all that apply.) a. Vitiligo b. Alopecia c. Intertrigo d. Erythema e. Actinic keratosis

D, E

What does the cell mediated immunity protect against?

Fungus, Viruses (intracellular), chronic infectious agents, tumor cells EX. TB, cancer cells

Which IG molecule can cross the placental membrane and provide newborns with passive acquired immunity?

IgG

What is a product of cell mediated immunity?

Sensitized T cells, cytokines

Which lymphocyte is part of cell mediated immunity?

T Lymphocytes

Effects of aging on the immune response

Thymic shrinkage Decreased cell mediated immunity

Anergy

absence of the normal immune response to a particular antigen or allergen

Type I, II, and III hypersensitivities are mediated by a. antibodies b. mast cells c. T lymphocytes d. macrophages e. eosinophils

antibodies

What is a product of humoral immunity?

antibodies

what does the humoral immunity protect against?

bacteria, extracellular viruses, respiratory and GI pathogens EX. Anaphylactic shock

To evaluate the effectiveness of antiretroviral therapy (ART), which laboratory test result will the nurse review? a. Viral load testing b. Enzyme immunoassay c. Rapid HIV antibody testing d. Immunofluorescence assay

A

A patient with a right lower leg fracture will be discharged home with an external fixation device in place. Which statement should the nurse including in discharge teaching? a. "Check and clean the pin insertion sites daily." b. "Remove the external fixator for your shower." c. "Remain on bed rest until bone healing is complete." d. "Take prophylactic antibiotics until the fixator is removed."

A

Assessment of a patient's visual acuity reveals that the left eye can see at 20 feet what a person with normal vision can see at 50 feet and the right eye can see at 20 feet what a person with normal vision can see at 40 feet. Which finding should the nurse record? a. OS 20/50; OD 20/40 b. OU 20/40; OS 50/20 c. OD 20/40; OS 20/50 d. OU 40/20; OD 50/20

A

The nurse is advising a patient who was exposed 4 days ago to human immunodeficiency virus (HIV) through unprotected sexual intercourse. The patient's antigen-antibody test has just been reported as negative for HIV. What information should the nurse give to this patient? a. "You will need to be retested in 2 weeks." b. "You do not need to fear infecting others." c. "We won't know for about 10 years if you have HIV infection." d. "With no symptoms and this negative test, you do not have HIV."

A

When caring for a patient who is pancytopenic, which action by unlicensed assistive personnel (UAP) indicates a need for the nurse to intervene? a. The UAP assists the patient to use dental floss after eating. b. The UAP adds baking soda to the patient's saline oral rinses. c. The UAP puts fluoride toothpaste on the patient's toothbrush. d. The UAP has the patient rinse after meals with a saline solution.

A

Which result for a patient with systemic lupus erythematosus (SLE) is most important for the nurse to communicate to the health care provider? A. Elevated blood urea nitrogen (BUN) B. Positive antinuclear antibodies (ANA) C. Positive lupus erythematosus cell prep D. Decreased C-reactive protein (CRP)

A

What features of cancer cells distinguish them from normal cells (select all that apply)? a. Cells lack contact inhibition. b. Cells undergo rapid proliferation. c. Cells return to a previous undifferentiated state. d. Proliferation occurs when there is a need for more cells. e. New proteins characteristic of embryonic stage emerge on cell membrane.

A, C, E

What should the nurse teach the patient before fluorescein angiography? a. Hold a card and fixate on the center dot. b. Report any burning or pain at the IV site. c. Remain still while the cornea is anesthetized. d. Let the examiner know when images shown appear clear.

B

Which patient would benefit from education about HIV preexposure prophylaxis (PrEP)? a. A 23-yr-old woman living with HIV infection. b. A 52-yr-old recently single woman just diagnosed with chlamydia. c. A 33-yr-old hospice worker who received a needle stick injury 3 hours ago. d. A 60-yr-old male in a monogamous relationship with an HIV-uninfected partner.

B

Which prescribed medication should the nurse give first to a patient who has just been admitted to a hospital with acute angle-closure glaucoma? a. Morphine sulfate 4 mg IV b. Mannitol (Osmitrol) 100 mg IV c. Betaxolol (Betoptic) 1 drop in each eye d. Acetazolamide (Diamox) 250 mg orally

B

A patient being seen in the clinic has rheumatoid nodules on the elbows. Which action should the nurse take? a. Draw blood for rheumatoid factor analysis. b. Teach the patient about injections for the nodules. c. Assess the nodules for skin breakdown or infection. d. Discuss the need for surgical removal of the nodules.

C

A patient treated for human immunodeficiency virus (HIV) infection for 6 years has developed fat redistribution to the trunk with wasting of the arms, legs, and face. What recommendation will the nurse give to the patient? a. Review foods that are higher in protein. b. Teach about the benefits of daily exercise. c. Discuss a change in antiretroviral therapy. d. Talk about treatment with antifungal agents.

C

A patient who is receiving immunotherapy has just received an allergen injection. Which assessment finding is most important to communicate to the health care provider? a. The patient's IgG level is increased. b. The injection site is red and swollen. c. There is a 2-cm wheal at the site of the injection. d. The patient's symptoms did not improve in 2 months.

C

The health care provider asks the nurse to evaluate whether a patient's angioedema has responded to prescribed therapies. Which assessment should the nurse perform? a. Obtain the patient's blood pressure and heart rate. b. Question the patient about any clear nasal discharge. c. Observe for swelling of the patient's lips and tongue. d. Assess the patient's extremities for wheal and flare lesions.

C

The nurse notices a circular lesion with a red border and clear center on the arm of a patient who is in the clinic reporting chills and muscle aches. Which action should the nurse take to follow up on that finding? a. Auscultate the heart sounds. b. Palpate the abdomen for masses. c. Ask the patient about recent outdoor activities. d. Question the patient about immunization history.

C

Which abnormality on the skin of an older patient is the priority for the nurse to discuss with the health care provider? a. Dry, scaly patches on the face b. Numerous varicosities on both legs c. Petechiae on the chest and abdomen d. Small dilated blood vessels on the face

C

Which information should the nurse include when teaching patients about decreasing the risk for sun damage to the skin? a. Use a sunscreen with an SPF of at least 10 for adequate protection. b. Water-resistant sunscreens provide good protection when swimming. c. Try to stay out of the direct sun between the hours of 10 AM and 2 PM. d. Increase sun exposure by no more than 10 minutes a day to avoid skin damage.

C

Which information will the nurse include when teaching an older patient about skin care? a. Dry the skin thoroughly before applying lotions. b. Bathe and wash hair daily with soap and shampoo. c. Use warm water and a moisturizing soap when bathing. d. Use antibacterial soaps when bathing to avoid infection.

C

Which laboratory result will the nurse monitor to determine if prednisone has been effective for a patient with an acute exacerbation of rheumatoid arthritis? A. Blood glucose B. Serum electrolytes C. C-reactive protein D. Liver function tests

C

A patient who has a positive test for human immunodeficiency virus (HIV) antibodies is admitted to the hospital with Pneumocystis jiroveci pneumonia (PCP) and a CD4+ count of less than 200 cells/μL. Based on diagnostic criteria established by the Centers for Disease Control and Prevention (CDC), which statement by the nurse is correct? a. "The patient meets the criteria for a diagnosis of acute HIV infection." b. "The patient will be diagnosed with asymptomatic chronic HIV infection." c. "The patient will likely develop symptomatic HIV infection within 1 year." d. "The patient has developed acquired immunodeficiency syndrome (AIDS)."

D

A patient who is human immunodeficiency virus (HIV)-infected has a CD4+ cell count of 400/μL. Which factor is most important for the nurse to determine before the initiation of antiretroviral therapy (ART) for this patient? a. CD4+ cell count b. How the patient obtained HIV c. Patient's tolerance for potential medication side effects d. Patient's ability to follow a complex medication regimen

D

A patient with an enlarging, irregular mole that is 7 mm in diameter is scheduled for outpatient treatment. The nurse should plan to prepare the patient for which procedure? a. Curettage b. Cryosurgery c. Punch biopsy d. Surgical excision

D

The nurse is caring for a 59-year-old woman who had surgery 1 day ago to remove an ovarian cancer mass. The patient is awaiting the pathology report. She is tearful and says that she is scared to die. The most effective nursing intervention at this point is to use this opportunity to a. motivate change in an unhealthy lifestyle. b. teach her about the 7 warning signs of cancer. c. discuss healthy stress relief and coping practices. d. let her communicate about the meaning of this experience.

D

The nurse is caring for a patient diagnosed with furunculosis. Which action could the nurse delegate to unlicensed assistive personnel (UAP)? a. Applying antibiotic cream to the groin b. Obtaining cultures from ruptured lesions c. Evaluating the patient's personal hygiene d. Cleaning the skin with antimicrobial soap

D

To decrease the risk for future hearing loss, which action should the nurse implement with college students at the on-campus health clinic? a. Perform tympanometry. b. Schedule otoscopic examinations. c. Administer influenza immunizations. d. Discuss exposure to amplified music.

D

Which patient seen by the nurse in the outpatient clinic is most likely to need teaching about ways to reduce the risk for osteoarthritis (OA)? a. A 56-yr-old man who has a sedentary office job b. A 38-yr-old man who plays on a summer softball team c. A 38-yr-old woman who is newly diagnosed with diabetes d. A 56-yr-old woman who works on an automotive assembly line

D

Which statement by a patient with discomfort from a bunion indicates to the nurse that more teaching is needed? a. "I will give away my high-heeled shoes." b. "I can take ibuprofen (Motrin) if I need it." c. "I will use the bunion pad to cushion the area." d. "I can only wear sandals, no closed-toe shoes."

D

A patient who had arthroscopic surgery of the right knee 7 days ago is admitted with a red, swollen, hot knee. Which assessment finding should the nurse report immediately to the health care provider? a. The blood pressure is 86/50 mm Hg. b. The patient says the knee pain is severe. c. The white blood cell count is 11,500/μL. d. The patient is taking ibuprofen (Motrin).

A

A patient who had open reduction and internal fixation (ORIF) of left lower leg fractures continues to report severe pain in the leg 15 minutes after receiving the prescribed IV morphine. The nurse determines pulses are faintly palpable and the foot is cool to the touch. Which action should the nurse take next? a. Notify the health care provider. b. Assess the incision for redness. c. Reposition the left leg on pillows. d. Check the patient's blood pressure.

A

Interleukin-2 (IL-2) is used as adjuvant therapy for a patient with metastatic renal cell carcinoma. Which information should the nurse include when explaining the purpose of this therapy to the patient? a. IL-2 enhances the body's immunologic response to tumor cells. b. IL-2 prevents bone marrow depression caused by chemotherapy. c. IL-2 protects normal cells from harmful effects of chemotherapy. d. IL-2 stimulates cancer cells in their resting phase to enter mitosis.

A

Which action should the nurse include when performing the straight-leg raising test for an ambulatory patient with back pain? a. Lift the patient's leg to a 60-degree angle from the bed. b. Place the patient in the prone position on the exam table. c. Ask the patient to dangle both legs over the edge of the exam table. d. Instruct the patient to elevate the legs and tense the abdominal muscles.

A

Which action should the nurse take to evaluate the effectiveness of Buck's traction for a patient who has an intracapsular fracture of the right femur? a. Assess for hip pain. b. Check for contractures. c. Palpate peripheral pulses. d. Monitor for hip dislocation.

A

Which action should the nurse take when caring for a patient who is receiving chemotherapy and reports problems with concentration? a. Suggest use of a daily planner and encourage adequate sleep. b. Teach the patient to rest the brain by avoiding new activities. c. Teach that "chemo-brain" is a short-term effect of chemotherapy. d. Report patient symptoms immediately to the health care provider.

A

Which action should the nurse take when caring for a patient with osteomalacia? a. Teach about the use of vitamin D supplements. b. Educate about the need for weight-bearing exercise. c. Instruct the patient to avoid dairy products in the diet. d. Discuss the use of medications such as bisphosphonates.

A

Which assessments by the nurse would indicate a greater risk to develop a nonmelanoma skin cancer? a. blond hair, freckles, fair skin b. alopecia, thin hair, itching c. dark hair, dark skin, dry skin d. tanned skin, dark hair, edema

A

Which finding for a patient who is taking hydroxychloroquine (Plaquenil) to treat rheumatoid arthritis should the nurse identify as a likely adverse effect of the medication? a. Blurred vision b. Joint tenderness c. Abdominal cramping d. Elevated blood pressure

A

Which finding from analysis of fluid from a patient's right knee arthrocentesis should be of concern to the nurse? a. Cloudy fluid b. Scant thin fluid c. Pale yellow fluid d. Straw-colored fluid

A

A patient is admitted to the emergency department with a left femur fracture. Which assessment finding by the nurse is most important to report to the health care provider? a. Bruising of the left thigh b. Reports of severe thigh pain c. Slow capillary refill of the left foot d. Outward pointing toes on the left foot

C

A patient is anxious and reports difficulty breathing after being stung by a wasp. What is the nurse's priority action? a. Provide high-flow oxygen. b. Administer antihistamines. c. Assess the patient's airway. d. Remove the stinger from the site.

C

The bone cells that function in the formation of new bone tissue are called a. osteoids. b. osteocytes. c. osteoclasts. d. osteoblasts.

D

A patient is undergoing psoralen plus ultraviolet A light (PUVA) therapy for treatment of psoriasis. What action should the nurse take to prevent adverse effects from this procedure? a. Shield any unaffected areas with lead-lined drapes. b. Apply petroleum jelly to the areas around the lesions. c. Cleanse the skin carefully with antiseptic soap prior to PUVA. d. Have the patient use protective eyewear while receiving PUVA.

D

A patient with age-related macular degeneration (AMD) has just had photodynamic therapy. Which statement by the patient indicates that the discharge teaching has been effective? a. "I will use drops to keep my pupils dilated until my appointment." b. "I will need to use brighter lights to read for at least the next week." c. "I will not use facial lotions near my eyes during the recovery period." d. "I will cover up with long-sleeved shirts and pants for the next 5 days."

D

A patient with human immunodeficiency virus (HIV) infection has developed Cryptosporidium parvum infection. Which outcome would be appropriate for the nurse to include in the plan of care? a. The patient will be free from injury. b. The patient will receive immunizations. c. The patient will have adequate oxygenation. d. The patient will maintain intact perineal skin.

D

A young adult arrives in the emergency department with ankle swelling and severe pain after twisting an ankle playing basketball. Which prescribed action will the nurse implement first? a. Send the patient for ankle x-rays. b. Administer naproxen (Naprosyn). c. Give acetaminophen with codeine. d. Wrap the ankle and apply an ice pack.

D

Association between HLA antigens and diseases is most commonly found in what disease conditions? a. Cancers b. Infectious diseases c. Neurologic diseases d. Autoimmune disorders

D

The function of monocytes in immunity is related to their ability to a. stimulate the production of T and B lymphocytes. b. make antibodies after exposure to foreign substances. c. bind antigens and stimulate natural killer cell activation. d. capture antigens by phagocytosis and present them to lymphocytes.

D

The home health nurse is making a follow-up visit to a patient recently diagnosed with rheumatoid arthritis (RA). Which finding indicates to the nurse that additional patient teaching is needed? a. The patient takes a 2-hour nap each day. b. The patient has been taking 16 aspirins each day. c. The patient sits on a stool while preparing meals. d. The patient sleeps with two pillows under the head.

D

Persons with dark skin are more likely to develop : a. keloids. b. wrinkles. c. skin rashes. d. skin cancer.

A

Which action should the nurse take before administering gentamicin (Garamycin) to a patient with acute osteomyelitis? a. Ask the patient about any nausea. b. Obtain the patient's oral temperature. c. Change the prescribed wet-to-dry dressings. d. Review the patient's serum creatinine results.

D

A patient who has had open reduction and internal fixation (ORIF) of a hip fracture tells the nurse he is ready to get out of bed for the first time. Which action should the nurse take? a. Check the patient's prescribed weight-bearing status. b. Use a mechanical lift to transfer the patient to the chair. c. Decrease the pain medication before getting the patient up. d. Have the unlicensed assistive personnel (UAP) transfer the patient.

A

A 65-yr-old patient is being evaluated for glaucoma. Which information given by the patient has implications for the patient's treatment plan? a. "I take metoprolol (Lopressor) for angina." b. "I take aspirin when I have a sinus headache." c. "I have had frequent episodes of conjunctivitis." d. "I have not had an eye examination for 10 years."

A

A mother and her two children have been diagnosed with pediculosis corporis at a health care center. An appropriate measure to treat this condition is a. applying pyrethrins to the body. b. topical application of griseofulvin. c. moist compresses applied frequently. d. administration of systemic antibiotics.

A

A nurse assesses the following common skin lesions. Which lesion has the potential of becoming malignant? a. nevi b. angiomas c. skin tags d. keloids

A

A patient has a new order for magnetic resonance imaging (MRI) to evaluate possible left femur osteomyelitis after hip arthroplasty surgery. Which information indicates the nurse should consult with the health care provider before scheduling the MRI? a. The patient has a pacemaker. b. The patient wears a hearing aid. c. The patient is allergic to shellfish. d. The patient uses supplemental oxygen.

A

A patient in the dermatology clinic has a thin, scaly erythematous plaque on the right cheek. Which action should the nurse take? a. Prepare the patient for a skin biopsy. b. Teach the use of corticosteroid cream. c. Explain how to apply tretinoin (Retin-A) to the face. d. Discuss the need for topical application of antibiotics.

A

A patient undergoes left above-the-knee amputation with an immediate prosthetic fitting. What should the nurse do when the patient arrives on the orthopedic unit after surgery? a. Assess the surgical site for hemorrhage. b. Remove the prosthesis and wrap the site. c. Place the patient in a side-lying position. d. Keep the residual limb elevated on a pillow.

A

A patient who has Ménière's disease is admitted with vertigo, nausea, and vomiting. Which nursing intervention will be included in the care plan? a. Dim the lights in the patient's room. b. Encourage increased oral fluid intake. c. Change the patient's position every 2 hours. d. Keep the head of the bed elevated 45 degrees.

A

A patient who is diagnosed with cervical cancer classified as Tis, N0, M0 asks the nurse what the letters and numbers mean. Which response by the nurse is accurate? a. "The cancer involves only the cervix." b. "The cancer cells look like normal cells." c. "Further testing is needed to determine the spread of the cancer." d. "It is difficult to determine the original site of the cervical cancer."

A

The nurse is assessing a 65-yr-old patient for presbyopia. Which instruction will the nurse give the patient before the test? a. "Hold this card and read the print out loud." b. "Cover one eye while reading the wall chart." c. "You'll feel a short burst of air directed at your eyeball." d. "A light will be used to look for a change in your pupils."

A

The nurse is assessing a patient who was recently treated with amoxicillin for acute otitis media of the right ear. Which finding is a priority to report to the health care provider? a. The patient has a temperature of 100.6° F. b. The patient report frequent "popping" in the ear. c. Clear fluid is visible through the tympanic membrane. d. The patient frequently asks the nurse to repeat information.

A

The nurse at the clinic is interviewing a 64-yr-old woman who is 5 feet, 3 inches tall and weighs 125 lb (57 kg). The patient has not seen a health care provider for 20 years. She walks 5 miles most days and has a glass of wine 2 or 3 times a week. Which topics will the nurse plan to include in patient teaching about cancer screening and decreasing cancer risk? (Select all that apply.) a. Pap testing b. Tobacco use c. Sunscreen use d. Mammography e. Colorectal screening

A, C, D, E

A patient who is receiving an IV antibiotic develops wheezes and dyspnea. In which order should the nurse implement these prescribed actions? (Put a comma and a space between each answer choice [A, B, C, D, E]). a. Discontinue the antibiotic. b. Give diphenhydramine IV. c. Inject epinephrine IM or IV. d. Prepare an infusion of dopamine. e. Apply 100% oxygen using a nonrebreather mask.

A, E, C, B, D

The increased risk for falls in the older adult is likely due to (select all that apply) a. changes in balance. b. decrease in bone mass. c. loss of ligament elasticity. d. erosion of articular cartilage. e. decrease in muscle mass and strength.

A, b, c, e

The nurse is caring for a patient living with asymptomatic chronic HIV infection (HIV). Which prophylactic measures will the nurse include in the plan of care? (Select all that apply.) a. Hepatitis B vaccine b. Pneumococcal vaccine c. Influenza virus vaccine d. Trimethoprim-sulfamethoxazole e. Varicella zoster immune globulin

A,B,C

The nurse plans a presentation for community members about how to decrease the risk for antibiotic-resistant infections. Which information will the nurse include in the teaching plan? (Select all that apply.) a. Antibiotics may sometimes be prescribed to prevent infection. b. Continue taking antibiotics until all of the prescription is gone. c. Unused antibiotics that are more than a year old should be discarded. d. Antibiotics are effective in treating influenza associated with high fevers. e. Hand washing is effective in preventing many viral and bacterial infections.

A,B,E

A patient develops neutropenia after receiving chemotherapy. Which information about ways to prevent infection will the nurse include in the teaching plan? (Select all that apply.) a. Cook food thoroughly before eating. b. Choose low fiber, low residue foods. c. Avoid public transportation such as buses. d. Use rectal suppositories if needed for constipation. e. Talk to the oncologist before having any dental work.

A,C,E

A patient is scheduled for a bone scan. The nurse explains that this diagnostic test involves a. incision or puncture of the joint capsule. b. insertion of small needles into certain muscles. c. administration of a radioisotope before the procedure. d. placement of skin electrodes to record muscle activity.

C

A hospitalized patient who has received chemotherapy for leukemia develops neutropenia. Which observation by the nurse indicates a need for further teaching? a. The patient ambulates around the room. b. The patient's visitors bring in fresh peaches. c. The patient cleans with a warm washcloth after having a stool. d. The patient uses soap and shampoo to shower every other day.

B

A nurse who works on the orthopedic unit has just received change-of-shift report. Which patient should the nurse assess first? a. Patient who reports foot pain after hammertoe surgery. b. Patient who has not voided 8 hours after a laminectomy. c. Patient with low back pain and a positive straight-leg-raise test. d. Patient with osteomyelitis who has a temperature of 100.5° F (38.1° C).

B

A patient with acute osteomyelitis of the left femur is hospitalized for regional antibiotic irrigation. Which intervention should the nurse include in the initial plan of care? a. Quadriceps-setting exercises b. Immobilization of the left leg c. Positioning the left leg in flexion d. Assisted weight-bearing ambulation

B

A clinic patient reports experiencing an allergic reaction to an unknown allergen several weeks ago. Which action is appropriate for the registered nurse (RN) to delegate to a licensed practical/vocational nurse (LPN/VN)? a. Perform a focused physical assessment. b. Administer a cutaneous scratch skin test. c. Obtain the health history from the patient. d. Review diagnostic study results with the patient.

B

What is the chief complaint by the patient with a diagnosis of gout? A. "I have pain in my right heal" B. "I woke up with severe pain in my big toe" C. "I woke up with pain in my lower spine" D. 'I bought new shoes now my toenail hurts"

B

A 60-yr-old patient had open reduction and internal fixation (ORIF) for an open, displaced tibial fracture, What should the nurse identify as the priority patient problem? a. Acute pain b. Risk for infection c. Activity intolerance d. Risk for constipation

B

A 75-yr-old patient who lives alone at home tells the nurse, "I am afraid of losing my independence because my eyes don't work as well they used to." Which action should the nurse take first? a. Discuss the increased risk for falls that is associated with impaired vision. b. Ask the patient about what type of vision problems are being experienced. c. Explain that there are many ways to compensate for decreases in visual acuity. d. Suggest ways of improving the patient's safety, such as using brighter lighting.

B

A chemotherapy drug that causes alopecia is prescribed for a patient. Which action should the nurse take to support the patient's self-esteem? a. Suggest that the patient limit social contacts until hair regrowth occurs. b. Encourage the patient to purchase a wig or hat to wear when hair loss begins. c. Teach the patient to wash hair gently with mild shampoo to minimize hair loss. d. Inform the patient that hair usually grows back once chemotherapy is complete.

B

Which information about a patient with a lumbar vertebral compression fracture should the nurse immediately report to the health care provider? a. Patient declines to be turned due to back pain. b. Patient has been incontinent of urine and stool. c. Patient reports lumbar area tenderness to palpation. d. Patient frequently uses oral corticosteroids to treat asthma.

B

In teaching a patient who is using topical corticosteroids to treat acute dermatitis, the nurse should tell the patient that (select all that apply) a. the cream form is the most efficient system of delivery. b. short-term use of topical corticosteroids usually does not cause systemic side effects. c. use a glove to apply small amounts of creams or ointments to prevent further infection. d. abruptly stopping the use of topical corticosteroids may cause the dermatitis to reappear. e. systemic side effects from topical corticosteroids are likely if the patient is malnourished.

B, D

Whatsignsandsymptomwouldyoufindina patient with a cataract? Select all that apply A. Increase in visual acuity B. Decrease in visual acuity C. Complaints of double vision D. Abnormal color perception E. Complaints of glared vision

B, D, E

According to the Center for Disease Control and Prevention (CDC) guidelines, which personal protective equipment will the nurse put on before assessing a patient who is on contact precautions for Clostridium difficile diarrhea? (Select all that apply.) a. Mask b. Gown c. Gloves d. Shoe covers e. Eye protection

B,C

A nurse is teaching a patient with contact dermatitis of the arms and legs about ways to decrease pruritus. Which information should the nurse include in the teaching plan? (Select all that apply.) a. Add oil to your bath water to moisturize the affected skin. b. Cool, wet clothes or compresses can be used to reduce itching. c. Use an over-the-counter (OTC) antihistamine to reduce itching. d. Take cool or tepid baths several times daily to decrease itching. e. Rub yourself dry with a towel after bathing to prevent skin maceration.

B,C,D

In a person having an acute rejection of a transplanted kidney, what would help the nurse understand the course of events (select all that apply)? a. A new transplant should be considered. b. Acute rejection can be treated with OKT3. c. Repeated episodes of acute rejection can lead to chronic rejection. d. Corticosteroids are the most successful drugs used to treat acute rejection. e. Acute rejection is common after a transplant and can be

B,C,E

A high school teacher with ulnar drift caused by rheumatoid arthritis (RA) is scheduled for arthroplasty of several joints in the left hand. Which patient statement to the nurse indicates a realistic expectation for the surgery? a. "This procedure will correct the deformities in my fingers." b. "I will not have to do as many hand exercises after the surgery." c. "I will be able to use my fingers with more flexibility to grasp things." d. "My fingers will appear more normal in size and shape after this surgery."

C

A nurse has obtained donor tissue typing information about a patient who is waiting for a kidney transplant. Which results should be reported to the transplant surgeon? a. Patient is Rh positive and donor is Rh negative. b. Six antigen matches are present in HLA typing. c. Results of patient-donor crossmatching are positive. d. Panel of reactive antibodies (PRA) percentage is low.

C

A nurse is caring for a young adult with acne scars. The patient asks about treatment to reduce the scarring. Which of the following might the nurse discuss? a. liposuction b. skin flap c. dermabrasion d. blepharoplasty

C

A patient is taking methotrexate to treat rheumatoid arthritis (RA). Which laboratory result is important for the nurse to communicate to the health care provider? a. Rheumatoid factor is positive. b. Fasting blood glucose is 90 mg/dL. c. The white blood cell count is 1500/μL. d. The erythrocyte sedimentation rate is increased.

C

A patient is undergoing plasmapheresis for treatment of systemic lupus erythematosus. The nurse explains that plasmapheresis is used in treatment to a. remove T lymphocytes in her blood that are producing antinuclear antibodies. b. remove normal particles in her blood that are being damaged by autoantibodies. c. exchange her plasma that contains antinuclear antibodies with a substitute fluid. d. replace viral-damaged cellular components of her blood with replacement whole blood.

C

External-beam radiation is planned for a patient with cervical cancer. What instructions should the nurse give to the patient to prevent complications from the effects of the radiation? a. Test all stools for the presence of blood. b. Maintain a high-residue, high-fiber diet. c. Clean the perianal area carefully after every bowel movement. d. Inspect the mouth and throat daily for the appearance of thrush.

C

On inspection of a patient's dark skin, the nurse notes a blue-gray birthmark on the forehead and eye area. This assessment finding is called a. vitiligo. b. intertrigo. c. Nevus of Ota. d. telangiectasia.

C

The nurse recording health histories in the outpatient clinic would plan a focused hearing assessment for adult patients taking which medication? a. Atenolol b. Albuterol c. Ibuprofen d. Acetaminophen

C

What should the nurse include when teaching older adults at a community recreation center about ways to prevent fractures? a. Tack down scatter rugs on the floor in the home. b. Expect most falls to happen outside the home in the yard. c. Buy shoes that provide good support and are comfortable to wear. d. Get instruction in range-of-motion exercises from a physical therapist.

C

Which statement by the patient to the home health nurse indicates a need for further teaching about self-administering eardrops? a. "I will leave the ear wick in place while administering the drops." b. "I will hold the tip of the dropper above the ear to administer the drops." c. "I will refrigerate the medication until I am ready to administer the drops." d. "I should lie down before and for 5 minutes after administering the drops."

C

hich information in a 67-yr-old woman's health history should alert the nurse to the need for a focused assessment of the musculoskeletal system? a. The patient sprained her ankle at age 13. b. The patient's father died of tuberculosis. c. The patient's mother became shorter with aging. d. The patient takes ibuprofen for occasional headaches.

C

A patient who has severe refractory psoriasis on the face, neck, and extremities is socially withdrawn because of the appearance of the lesions. Which action should the nurse take first? a. Discuss the possibility of taking part in an online support group. b. Encourage the patient to volunteer to work on community projects. c. Suggest that the patient use cosmetics to cover the psoriatic lesions. d. Ask the patient to describe the impact of psoriasis on quality of life.

D

A patient with osteomyelitis is treated with surgical debridement followed by continuous irrigation of the affected bone with antibiotics. In responding to the patient who asks why oral or IV antibiotics cannot be used alone, the nurse explains that A. The irrigation is necessary to wash out dead tissue and pus from the infected area B. There are no effective oral or IV antibiotics the treat S. aureus C. An irrigation can penetrate involucrum created by the infection and prevent bacterial spreading to other tissue. D. The ischemia and bone death associated with osteomyelitis are frequently impenetrable to most blood- borne antibiotics.

D

An older adult patient who has colorectal cancer is receiving IV fluids at 175 mL/hr in conjunction with the prescribed chemotherapy. Which finding by the nurse is most important to report to the health care provider? a. Patient reports having severe fatigue. b. Patient voids every hour during the day. c. Patient takes only 50% of meals and refuses snacks. d. Patient has crackles up to the midline posterior chest.

D

In a type I hypersensitivity reaction the primary immunologic disorder appears to be a. binding of IgG to an antigen on a cell surface. b. deposit of antigen-antibody complexes in small vessels. c. release of cytokines used to interact with specific antigens. d. release of chemical mediators from IgE-bound mast cells and basophils.

D

The nurse in the eye clinic is examining a 67-yr-old patient who says, "I see small spots that move around in front of my eyes." Which action will the nurse take first? a. Immediately have the ophthalmologist evaluate the patient. b. Explain that spots and "floaters" are a normal part of aging. c. Warn the patient that these spots may indicate retinal damage. d. Use an ophthalmoscope to examine the posterior eye chambers.

D

The nurse is assessing the skin, what assessment skill will be used? A.Inspection & Percussion B. Percussion & Auscultation C. Auscultation & Palpation D. Palpation & Inspection

D

Which finding by the nurse performing an eye examination indicates that the patient has normal accommodation? a. After covering one eye for 1 minute, the pupil constricts as the cover is removed. b. Shining a light into the patient's eye causes pupil constriction in the opposite eye. c. A blink reaction occurs after touching the patient's pupil with a piece of sterile cotton. d. The pupils constrict while fixating on an object being moved toward the patient's eyes.

D

Which finding for a 77-yr-old patient seen in the outpatient clinic is the highest priority for further nursing assessment and intervention? a. Symmetric joint swelling of fingers b. Decreased right knee range of motion c. Report of left hip aching when jogging d. History of recent loss of balance and fall

D

Which information will the nurse include for a patient considering a cochlear implant? Cochlear implants: a. are not useful for patients with congenital deafness. b. are most helpful as an early intervention for presbycusis. c. improve hearing in patients with conductive hearing loss. d. require extensive training in order to reach the full benefit.

D

Which teaching point should the nurse plan to include when caring for a patient whose vision is corrected to 20/200? a. How to access audio books? b. How to use a white cane safely? c. Where Braille instruction is available? d. Where to obtain hand-held magnifiers?

D

What is the priority problem for a patient experiencing an acute attack with Meniere's disease? a. Being at risk for falls b. Imbalanced nutritional intake c. Difficulty performing self-care d. Impaired verbal communication

A

In identifying people at risk for back injuries, the nurse recognizes that the person at greatest risk for low back pain is a(n) A. Long-distancetruckdriver B. 62-year-oldwidowwhowalksdaily C. Aerobicsinstructorwhoweighs100lb D. 25-year-oldnursewhoworksinanewborn nursery

A

What is a potential complication of both osteoporosis and osteomalacia? A. Infection B. Bloodclots C. Fractures D. Contractures

C

During assessment of the patient's skin, the nurse observes a similar pattern of discrete, small, raised lesions on the left and right upper back areas. Which term should the nurse use to document the distribution of these lesions? a. Confluent b. Symmetric c. Zosteriform d. Generalized

B

During the assessment of a patient, you note an area of red, sharply defined plaques covered with silvery scales that are mildly itchy on the patient's knees and elbows. You would describe this finding as a. lentigo. b. psoriasis. c. actinic keratosis. d. seborrheic keratosis.

B

The primary function of the skin is : a. insulation. b. protection. c. sensation. d. absorption.

B

Which of the following skin lesions, if assessed, is a risk for skin cancer? a. lice infestation b. actinic keratosis c. pressure ulcer d. folliculitis

B

A patient who underwent eye surgery must wear an eye patch until the scheduled postoperative clinic visit. Which patient problem will the nurse address in the plan of care? a. Risk for falls b. Difficulty coping c. Disturbed body image d. Inability to care for home

A

A patient with a torn ligament in the knee asks what the ligament does. The nurse's response is based on the knowledge that ligaments a. connect bone to bone. b. provide strength to muscle. c. lubricate joints with synovial fluid. d. relieve friction between moving parts.

A

A patient with left knee pain is diagnosed with bursitis. What area should the nurse explain is the site of inflammation in bursitis? a. A fluid-filled sac found at some joints. b. A synovial membrane that lines some joints. c. The connective tissue joining bones within a joint. d. The fibrocartilage that acts as a shock absorber in the knee.

A

An abnormal assessment finding of the musculoskeletal system is a. equal leg length bilaterally. b. ulnar deviation and subluxation. c. full range of motion in all joints. d. muscle strength of 5/5 in all muscles.

B

When teaching a patient with melanoma about this disorder, the nurse recognizes that the patient's prognosis is most dependent on a. the thickness of the lesion. b. the degree of asymmetry in the lesion. c. the amount of ulceration in the lesion. d. how much the lesion has spread superficially.

A

The primary protective role of the immune system related to malignant cells is a. surveillance for cells with tumor-associated antigens. b. binding with free antigens released by all cancer cells. c. producing blocking factors that immobilize cancer cells. d. reacting to a new set of antigenic determinants on cancer cells.

A

A patient with severe kyphosis is scheduled for dual-energy x-ray absorptiometry (DXA) testing. Which action should the nurse plan to take? a. Explain the procedure to the patient. b. Start an IV line for contrast injection. c. Give an oral sedative 60 to 90 minutes before the procedure. d. Screen the patient for allergies to shellfish or iodine products.

A

After teaching a 28-yr-old with fibromyalgia about the disease, which patient statement does the nurse determines indicates a good understanding of effective self-management? a. "I will need to stop drinking so much coffee and soda." b. "I am going to join a soccer team to get more exercise." c. "I will call the doctor every time my symptoms get worse." d. "I should avoid using over-the-counter medications for pain."

A

After the nurse has taught a 28-yr-old with fibromyalgia, which statement by the patient indicates a good understanding of effective self-management? A. "I will need to stop drinking so much coffee and soda." B. I am going to join a soccer team to get more exercise." C. "I will call the doctor every time my symptoms get worse." D. "I should avoid using over-the-counter medications for pain."

A

An older adult patient who is having an annual checkup tells the nurse, "I feel fine, and I don't want to pay for all these unnecessary cancer screening tests!" Which information should the nurse plan to teach this patient? a. Consequences of aging on cell-mediated immunity b. Decrease in antibody production associated with aging c. Incidence of cancer-associated infections in older adults d. Impact of poor nutrition on immune function in older adults

A

An older adult who takes medications for coronary artery disease and hypertension is newly diagnosed with HIV infection and is starting antiretroviral therapy. Which information will the nurse include in patient teaching? a. Many drugs interact with antiretroviral medications. b. HIV infections progress more rapidly in older adults. c. Less frequent CD4+ level monitoring is needed in older adults. d. Hospice care is available for patients with terminal HIV infection.

A

Eight years after seroconversion, a patient with human immunodeficiency virus infection has a CD4+ cell count of 800/μL and an undetectable viral load. What should be included in the plan of care at this time? a. Encourage adequate nutrition, exercise, and sleep. b. Teach about the side effects of antiretroviral agents. c. Explain opportunistic infections and antibiotic prophylaxis. d. Monitor symptoms of acquired immunodeficiency syndrome (AIDS).

A

For a patient who has had right hip arthroplasty, which nursing action can the nurse delegate to experienced unlicensed assistive personnel (UAP)? a. Reposition the patient every 1 to 2 hours. b. Assess for skin irritation on the patient's back. c. Teach the patient quadriceps-setting exercises. d. Determine the patient's pain intensity and tolerance.

A

Important patient teaching after a chemical peel includes : a. avoidance of sun exposure. b. application of firm bandages. c. limitation of vigorous exercise. d. use of moist heat to relieve discomfort.

A

Of the following, which assessment is most significant to the development of a melanoma? a. a change in the color or size of a nevus b. sexual contact with a person who has a herpes virus infection c. inadequate knowledge about infection prevention d. a dietary intake of high-calorie foods

A

The nurse assessed the patient's skin lesions as firm, edematous, irregularly shaped with a variable diameter. They would be called a. wheals. b. papules. c. pustules. d. plaques.

A

The nurse assesses a patient with non-Hodgkin's lymphoma who is receiving an infusion of rituximab (Rituxan). Which assessment finding would require the most rapid action by the nurse? a. Shortness of breath b. Shivering and chills c. Muscle aches and pains d. Temperature of 100.2° F (37.9° C)

A

The nurse is alerted to possible anaphylactic shock immediately after a patient has received IM penicillin by the development of a. edema and itching at the injection site. b. sneezing and itching of the nose and eyes. c. a wheal-and-flare reaction at the injection site. d. chest tightness and production of thick sputum.

A

The nurse is obtaining a health history of a patient with a fracture. Which condition poses the most concern related to the musculoskeletal system? a. Diabetes b. Hypertension c. Chronic bronchitis d. Nephrotic syndrome

A

The nurse is testing the visual acuity of a patient in the outpatient clinic. Which instructions should the nurse give for this test? a. "Stand 20 feet away from the wall chart." b. "Look at an object far away and then near to you." c. "Follow the examiner's finger with your eyes only." d. "Look straight ahead while I check your eyes with a light."

A

The nurse notes white lesions that resemble milk curds in the back of a patient's throat. Which question by the nurse is appropriate at this time? a. "Are you taking any medications?" b. "Do you have a productive cough?" c. "How often do you brush your teeth?" d. "Have you had an oral herpes infection?"

A

The nurse provides discharge instructions to a patient who has an immune deficiency involving the T lymphocytes. Which health screening should the nurse include in the teaching plan for this patient? a. Screening for cancers b. Screening for allergies c. Screening for antibody deficiencies d. Screening for autoimmune disorders

A

The nurse working in the clinic receives telephone calls from several patients who want appointments as soon as possible. Which patient should be seen first? a. 71-yr-old who has noticed increasing loss of peripheral vision b. 74-yr-old who has difficulty seeing well enough to drive at night c. 60-yr-old who is reporting dry eyes with decreased tear formation d. 64-yr-old who states that it is becoming difficult to read news print

A

The nurse working in the dermatology clinic assesses a young adult female patient who has severe cystic acne. Which assessment finding is of concern related to the patient's prescribed isotretinoin? a. The patient recently had an intrauterine device removed. b. The patient already has some acne scarring on her forehead. c. The patient has also used topical antibiotics to treat the acne. d. The patient has a strong family history of rheumatoid arthritis.

A

What finding should indicate to the nurse that colchicine has been effective for a patient with an acute attack of gout? a. Reduced joint pain b. Increased urine output c. Elevated serum uric acid d. Increased white blood cells

A

Which action should the nurse include in the plan of care for a patient with newly diagnosed ankylosing spondylitis? a. Have the patient sleep on their back with a flat pillow. b. Discuss that application of heat may worsen symptoms. c. Schedule annual laboratory assessment for the HLA-B27 antigen. d. Assist patient to choose physical activities that involve spinal flexion.

A

Which information about a patient population would be most useful to help the nurse plan for human immunodeficiency virus (HIV) testing needs? a. Age b. Lifestyle c. Symptoms d. Sexual orientation

A

Which information should the nurse include when teaching a patient who has just received a prescription for ciprofloxacin (Cipro) to treat a urinary tract infection? a. Use a sunscreen with a high SPF when exposed to the sun. b. Sun exposure may decrease the effectiveness of the medication. c. Photosensitivity may result in an artificial-looking tan appearance. d. Wear sunglasses to avoid eye damage while taking this medication.

A

Which instruction should the nurse include in a teaching plan for a patient with herpes simplex keratitis? a. Wash hands frequently and avoid touching the eyes. b. Apply antibiotic drops to the eye several times daily. c. Apply a new occlusive dressing to the affected eye at bedtime. d. Use corticosteroid ophthalmic ointment to decrease inflammation.

A

Which instructions should the nurse include in the teaching plan for a patient with impetigo? a. Clean the crusted areas with soap and water. b. Spread alcohol-based cleansers on the lesions. c. Avoid use of antibiotic ointments on the lesions. d. Use petroleum jelly (Vaseline) to soften crusty areas.

A

A patient who reports chronic itching of the ankles continuously scratches the area. Which assessment finding should the nurse expect? a. Hypertrophied scars on both ankles b. Thickening of the skin around the ankles c. Yellowish-brown skin around both ankles d. Complete absence of melanin in both ankles

B

A patient has recently been diagnosed with rheumatoid arthritis (RA) The patient, who has two school-age children, tells the nurse that home life is very stressful. Which initial response should the nurse make? a. "You need to see a family therapist for some help with stress." b. "Tell me more about the situations that are causing you stress." c. "Perhaps it would be helpful for your family to be in a support group." d. "Your family should understand the impact of your rheumatoid arthritis."

B

A patient has scleroderma manifested by CREST (calcinosis, Raynaud's phenomenon, esophageal dysfunction, sclerodactyly, and telangiectasia) syndrome. Which action should the nurse include in the plan of care? a. Avoid use of capsaicin cream on hands. b. Keep the environment warm and draft free. c. Obtain capillary blood glucose before meals. d. Assist to bathroom every 2 hours while awake.

B

A patient has the following risk factors for melanoma. Which risk factor should the nurse assign as the priority focus of patient teaching? a. The patient has multiple dysplastic nevi. b. The patient uses a tanning booth weekly. c. The patient is fair-skinned with blue eyes. d. The patient's mother died of malignant melanoma.

B

A patient hospitalized with polymyositis has joint pain; erythematous facial rash; eyelid edema; and a weak, hoarse voice. What safety priority should the nurse identify for this patient? a. Acute pain b. Risk for aspiration c. Impaired tissue integrity d. Disturbed visual perception

B

A patient in the health care provider's office for allergen testing using the cutaneous scratch method develops itching and swelling at the skin site. Which action should the nurse plan to take first? a. Monitor the patient's edema. b. Administer a dose of epinephrine. c. Obtain a prescription for oral antihistamines. d. Assess the patient's use of new skin products.

B

A patient is admitted to the hospital with acute rejection of a kidney transplant. Which intervention will the nurse expect for this patient? a. Testing for human leukocyte antigen (HLA) match b. Administration of immunosuppressant medications c. Insertion of an arteriovenous graft for hemodialysis d. Placement of the patient on the transplant waiting list

B

A patient is being discharged after 1 week of IV antibiotic therapy for acute osteomyelitis in the right leg. Which information should the nurse include in the discharge teaching? a. How to apply warm packs to the leg to reduce pain b. How to monitor and care for a long-term IV catheter c. The need for daily aerobic exercise to help maintain muscle strength d. The reason for taking oral antibiotics for 7 to 10 days after discharge

B

A patient is being evaluated for possible atopic dermatitis. The nurse should expect elevation of which laboratory value? a. IgA b. IgE c. Basophils d. Neutrophils

B

A patient reports dizziness when bending over and of nausea and dizziness associated with physical activities. What exam should the nurse expect to prepare the patient to undergo? a. Tympanometry b. Rotary chair testing c. Pure-tone audiometry d. Bone-conduction testing

B

A patient reports shoulder pain when the nurse moves the patient's arm behind the back. Which question should the nurse ask? a. "Are you able to feed yourself without difficulty?" b. "Do you have difficulty when you are putting on a shirt?" c. "Are you able to sleep through the night without waking?" d. "Do you ever have trouble lowering yourself to the toilet?"

B

A patient seeks care in the emergency department after sharing needles for heroin injection with a friend who has hepatitis B. To provide immediate protection from infection, what medication will the nurse expect to administer? a. Corticosteroids b. Gamma globulin c. Hepatitis B vaccine d. Fresh frozen plasma

B

A patient who arrives at the emergency department with severe left knee pain is diagnosed with a patellar dislocation. What should be the nurse's initial focus for patient teaching? a. Use of a knee immobilizer b. Monitored anesthesia care c. Physical activity restrictions d. Performance of gentle knee flexion

B

A patient who collects honey to earn supplemental income has developed a hypersensitivity to bee stings. Which statement by the patient would indicate a need for additional teaching? a. "I need to find a different way to earn extra money." b. "I will take oral antihistamines before going to work." c. "I can get a prescription for epinephrine and learn to self-inject it." d. "I should wear a Medic-Alert bracelet indicating my allergy to bee stings."

B

A patient who is being treated for stage IV lung cancer tells the nurse about new-onset back pain. Which action should the nurse take first? a. Give the patient the prescribed PRN opioid. b. Assess for sensation and strength in the legs. c. Notify the health care provider about the symptoms. d. Teach the patient how to use relaxation to reduce pain.

B

A patient whose employment requires frequent lifting has a history of chronic back pain. After the nurse has taught the patient about correct body mechanics, which patient statement indicates the teaching has been effective? a. "I will keep my back straight when I lift above than my waist." b. "I will begin doing exercises to strengthen and support my back." c. "I will tell my boss I need a job where I can stay seated at a desk." d. "I can sleep with my hips and knees extended to prevent back strain."

B

A patient with Hodgkin's lymphoma who is undergoing external radiation therapy tells the nurse, "I am so tired I can hardly get out of bed in the morning." Which intervention should the nurse add to the plan of care? a. Minimize activity until the treatment is completed. b. Establish time to take a short walk almost every day. c. Consult with a psychiatrist for treatment of depression. d. Arrange for delivery of a hospital bed to the patient's home.

B

A patient with a large stomach tumor attached to the liver is scheduled for a debulking procedure. What should the nurse teach the patient about the outcome of this procedure? a. Pain will be relieved by cutting sensory nerves in the stomach. b. Decreasing the tumor size will improve the effects of other therapy. c. Relieving the pressure in the stomach will promote optimal nutrition. d. Tumor growth will be controlled by removing all the cancerous tissue.

B

A patient with atopic dermatitis has a new prescription for pimecrolimus (Elidel). After teaching the patient about the medication, which statement by the patient indicates that further teaching is needed? a. "After I apply the medication, I can get dressed as usual." b. "If the medication burns when I apply it, I will wipe it off." c. "I need to minimize time in the sun while using the Elidel." d. "I will rub the medication in gently every morning and night."

B

A patient with glaucoma who has been using timolol (Timoptic) drops for several days tells the nurse that the eyedrops cause eye burning and visual blurriness after administration. The best response to the patient's statement is a. "Those symptoms may indicate a need for a change in dosage of the eyedrops." b. "The drops are uncomfortable, but it is important to use them to retain your vision." c. "These are normal side effects of the drug, which should be less noticeable with time." d. "Notify your health care provider so that different eyedrops can be prescribed for you."

B

A patient with gout has a new prescription for losartan (Cozaar). What should the nurse plan to monitor? a. Blood glucose b. Blood pressure c. Erythrocyte count d. Lymphocyte count

B

A patient with leukemia is considering whether to have hematopoietic stem cell transplantation (HSCT). Which information should the nurse include in the patient's teaching plan? a. Donor bone marrow is transplanted through a sternal or hip incision. b. Hospitalization is required for several weeks after the stem cell transplant. c. The transplant procedure takes place in a sterile operating room to decrease the risk for infection. d. Transplant of the donated cells can be very painful because of the nerves in the tissue lining the bone.

B

A patient with muscular dystrophy is hospitalized with pneumonia. Which nursing action should the nurse include in the plan of care? a. Logroll the patient every 2 hours. b. Assist the patient with ambulation. c. Discuss the need for genetic testing with the patient. d. Teach the patient about the muscle biopsy procedure.

B

A patient with presbycusis is fitted with binaural hearing aids. Which information will the nurse include when teaching the patient how to use the hearing aids? a. Keep the volume low on the hearing aids for the first week. b. Experiment with volume and hearing in a quiet environment. c. Add the second hearing aid after making adjustments to the first hearing aid. d. Begin wearing the hearing aids for an hour a day, gradually increasing the use.

B

A patient with psoriatic arthritis and back pain is receiving etanercept (Enbrel). Which finding is most important for the nurse to report to the health care provider? a. Red, scaly patches are noted on the arms. b. Crackles are auscultated in the lung bases. c. Hemoglobin is 11.1g/dL, and hematocrit is 35%. d. Patient has continued pain after first week of therapy.

B

A pregnant woman with asymptomatic chronic human immunodeficiency virus (HIV) infection is seen at the clinic. The patient states, "I am very nervous about making my baby sick." Which information will the nurse include when teaching the patient? a. The antiretroviral medications used to treat HIV infection are teratogenic. b. Most infants born to HIV-positive mothers are not infected with the virus. c. Because it is an early stage of HIV infection, the infant will not contract HIV. d. Her newborn will be born with HIV unless she uses antiretroviral therapy (ART).

B

A tennis player has an arthroscopic repair of a rotator cuff injury performed in same-day surgery. Which information will the nurse include in postoperative teaching? a. "You will not be able to serve a tennis ball again." b. "You will begin work with a physical therapist tomorrow." c. "Keep the shoulder immobilizer on for the first 4 days to minimize pain." d. "The surgeon will use the drop arm test to determine the success of surgery."

B

A widowed mother of 4 school-age children is hospitalized with metastatic ovarian cancer. The patient is crying and tells the nurse that she does not know what will happen to her children when she dies. Which response by the nurse is most appropriate? a. "Don't you have any friends that will raise the children for you?" b. "Would you like to talk about options for the care of your children?" c. "For now you need to concentrate on getting well and not worrying about your children." d. "Many patients with cancer live for a long time, so there is time to plan for your children."

B

After being hospitalized for 3 days with a right femur fracture, a patient suddenly develops shortness of breath and tachypnea. The patient tells the nurse, "I feel like I am going to die!" Which action should the nurse take first? a. Stay with the patient and offer reassurance. b. Administer prescribed PRN O2 at 4 L/min. c. Check the patient's legs for swelling or tenderness. d. Notify the health care provider about the symptoms.

B

After change-of-shift report, which patient should the nurse assess first? a. Patient with a repaired mandibular fracture who is reporting facial pain. b. Patient with repaired right femoral shaft fracture who reports tightness in the calf. c. Patient with an unrepaired Colles' fracture who has right wrist swelling and deformity. d. Patient with an unrepaired intracapsular left hip fracture whose leg is externally rotated.

B

Anakinra (Kineret) is prescribed for a patient with rheumatoid arthritis (RA). What information should the nurse include in teaching the patient about this drug? a. Avoiding aspirin use. b. Giving subcutaneous injections. c. Taking the medication with water. d. Recognizing gastrointestinal bleeding.

B

Before assisting a patient with ambulation 2 days after total hip arthroplasty, which action is most important for the nurse to take? a. Observe output from the surgical drain. b. Administer prescribed pain medication. c. Instruct the patient about benefits of early ambulation. d. Change the dressing and document the wound appearance.

B

Diagnostic testing is recommended for skin lesions when: a. a health history cannot be obtained. b. a more definitive diagnosis is needed. c. percussion reveals an abnormal finding. d. treatment with prescribed medication has failed.

B

During a routine health examination, a 40-yr-old patient tells the nurse about a family history of colon cancer. Which action should the nurse take next? a. Schedule a sigmoidoscopy to provide baseline data. b. Obtain more information about the patient's relatives. c. Teach the patient about the need for a colonoscopy at age 50. d. Teach the patient how to do home testing for fecal occult blood.

B

During the teaching session for a patient who has a new diagnosis of acute leukemia, the patient is restless and looks away without making eye contact. The patient asks the nurse to repeat the information about the complications associated with chemotherapy. Based on this assessment, which patient problem should the nurse identify? a. Denial b. Anxiety c. Acute confusion d. Ineffective adherence to treatment

B

How should the nurse evaluate a patient for improvement after treatment of primary open-angle glaucoma (POAG)? a. Question the patient about blurred vision. b. Note any changes in the patient's visual field. c. Ask the patient to rate the pain using a 0 to 10 scale. d. Assess the patient's depth perception when climbing stairs.

B

How should the nurse suggest that a patient recently diagnosed with rheumatoid arthritis (RA) plan to start each day? a. A brief routine of isometric exercises b. A warm bath followed by a short rest c. Active range-of-motion (ROM) exercises d. Stretching exercises to relieve joint stiffness

B

Immediately after the nurse administers an intradermal injection of an allergen on the forearm, the patient reports itching at the site, weakness, and dizziness. What action should the nurse take first? a. Apply antiinflammatory cream. b. Place a tourniquet above the site. c. Administer subcutaneous epinephrine. d. Reschedule the patient's other allergen tests.

B

The charge nurse observes a newly hired nurse performing all the following interventions for a patient who has just undergone right cataract removal and an intraocular lens implant. Which action requires that the charge nurse intervene? a. The nurse leaves the eye shield in place. b. The nurse encourages the patient to cough. c. The nurse elevates the patient's head to 45 degrees. d. The nurse applies corticosteroid drops to the right eye.

B

The day after a having a right below-the-knee amputation, a patient reports pain in the missing right foot. Which action is most important for the nurse to take? a. Explain the reasons for the pain. b. Administer prescribed analgesics. c. Reposition the patient to assure good alignment. d. Tell the patient that the pain will diminish over time.

B

The health care provider has prescribed the following interventions for a patient who is taking azathioprine (Imuran) for systemic lupus erythematosus. Which order should the nurse question? a. Draw anti-DNA blood titer. b. Administer varicella vaccine. C. Naproxen 200 mg twice daily D. Famotidine (Pepcid) 20 mg daily

B

The health care provider prescribes topical 5-FU for a patient with actinic keratosis on the left cheek. Which statement should the nurse include in the patient's instructions? a. "5-FU will shrink the lesion to prepare for surgical excision." b. "Your cheek area will be eroded and take several weeks to heal." c. "You may develop nausea and anorexia, but good nutrition is important during treatment." d. "You will need to avoid crowds because of the risk for infection caused by chemotherapy."

B

The home health nurse is caring for a patient who has been receiving interferon therapy for treatment of cancer. Which statement by the patient indicates a need for further assessment? a. "I have frequent muscle aches and pains." b. "I rarely have the energy to get out of bed." c. "I experience chills after I inject the interferon." d. "I take acetaminophen (Tylenol) every 4 hours."

B

The nurse administers an IV vesicant chemotherapeutic agent to a patient. Which action is most important for the nurse to take? a. Infuse the medication over a short period of time. b. Stop the infusion if swelling is observed at the site. c. Administer the chemotherapy through a small-bore catheter. d. Hold the medication unless a central venous line is available.

B

The nurse advises the patient with early osteoporosis to A. Loseweight B. Stopsmoking C. Eatahigh-proteindiet D. Startswimmingforexercise

B

The nurse assesses a circular, flat, reddened lesion about 5 cm in diameter on a middle-aged patient's ankle. How should the nurse determine if the lesion is related to intradermal bleeding? a. Elevate the patient's leg. b. Press firmly on the lesion. c. Check the temperature of the skin around the lesion. d. Palpate the dorsalis pedis and posterior tibial pulses.

B

The nurse assesses a patient who is receiving interleukin-2. Which finding should the nurse report immediately to the health care provider? a. Generalized muscle aches b. Crackles at the lung bases c. Reports of nausea and anorexia d. Oral temperature of 100.6° F (38.1° C)

B

The nurse determines that a patient with which disorder is most at risk for spreading the disease? a. Tinea pedis b. Impetigo on the face c. Candidiasis of the nails d. Psoriasis on the palms and soles

B

Unlicensed assistive personnel (UAP) perform the following actions when caring for a patient with Ménière's disease who is experiencing an acute attack. Which action by UAP indicates that the nurse should intervene? a. UAP raises the side rails on the bed. b. UAP turns on the patient's television. c. UAP places an emesis basin at the bedside. d. UAP helps the patient turn to the right side.

B

The nurse instructs a patient with primary open angle glaucoma about the disorder. Which of the following statements, if made by the nurse, is most appropriate? A. "The retinal nerve is damaged by an abnormal increase in the production of aqueous humor." B. "Aqueous humor cannot drain from the eye, causing pressure damage to the optic nerve." C. "As the lens enlarges with aging, it pushes the iris forward, covering the outflow channels of the eye." D. "The lens blocks the pupillary opening, preventing the flow of aqueous humor into the anterior chamber. "

B

The nurse is assessing a patient with osteoarthritis who uses naproxen (Naproxyn) for pain management. Which assessment finding should the nurse recognize as likely to require a change in medication? a. The patient has gained 3 pounds. b. The patient has dark-colored stools. c. The patient's pain affects multiple joints. d. The patient uses capsaicin cream (Zostrix).

B

The nurse is caring for a patient diagnosed with adult inclusion conjunctivitis (AIC) caused by C. trachomatis. Which action should be included in the plan of care? a. Applying topical corticosteroids to decrease inflammation b. Discussing the need for sexually transmitted infection testing c. Educating about the use of antiviral eyedrops to treat the infection d. Assisting with applying for community visual rehabilitation services

B

The nurse is caring for a patient who is using Buck's traction after a hip fracture. Which action can the nurse delegate to experienced unlicensed assistive personnel (UAP)? a. Remove and reapply traction periodically. b. Ensure the weight for the traction is hanging freely. c. Monitor the skin under the traction boot for redness. d. Check for intact sensation and movement in the affected leg.

B

The nurse is caring for a patient with colon cancer who is scheduled for external radiation therapy to the abdomen. Which information obtained by the nurse would indicate a need for patient teaching? a. The patient has a history of dental caries. b. The patient swims several days each week. c. The patient snacks frequently during the day. d. The patient showers each day with mild soap.

B

The nurse is observing a student who is preparing to perform an ear examination for a 30-year-old patient. Which action by the student indicates that the nurse should intervene? a. Pulls the auricle of the ear up and posterior. b. Chooses a speculum larger than the ear canal. c. Stabilizes the hand holding the otoscope on the patient's head. d. Stops inserting the otoscope after observing impacted cerumen.

B

The nurse is performing an eye examination on a 76-yr-old patient. Which finding indicates that the nurse should refer the patient for a more extensive assessment? a. The patient's sclerae are light yellow. b. The patient reports persistent photophobia. c. The pupil recovers slowly after responding to a bright light. d. There is a whitish gray ring encircling the periphery of the iris.

B

The nurse notes crackling sounds and a grating sensation with palpation of an older patient's elbow. How should this finding be documented? a. Torticollis b. Crepitation c. Subluxation d. Epicondylitis

B

The nurse should determine additional instruction is needed when a patient diagnosed with scleroderma makes which statement? a. "Paraffin baths can be used to help my hands." b. "I should lie down for an hour after each meal." c. "Lotions will help if I rub them in for a long time." d. "I should perform range-of-motion exercises daily."

B

The nurse should instruct a patient with a nondisplaced fractured left radius that the cast will need to remain in place for what amount of time? a. Two weeks b. At least six weeks c. Until swelling of the wrist has resolved d. Until x-rays show complete bony union

B

The nurse should suggest which food choice for a patient scheduled to receive external-beam radiation for abdominal cancer? a. Fruit salad b. Baked chicken c. Creamed broccoli d. Toasted wheat bread

B

The nurse taking a health history learns that the patient, who has worked in rubber tire manufacturing, has allergic rhinitis and multiple food allergies. Which action by the nurse is correct? a. Recommend that the patient use latex gloves in preventing blood-borne pathogen contact. b. Document the patient's history and teach about clinical manifestations of a type I latex allergy. c. Encourage the patient to carry an epinephrine kit in case a type IV allergic reaction to latex develops. d. Tell the patient to use oil-based hand creams to decrease contact with natural proteins in latex gloves.

B

The nurse teaches a patient about drug therapy after a kidney transplant. Which statement by the patient would indicate a need for further instructions? a. "I need to be monitored closely for development of cancer." b. "After a couple of years, I will be able to stop taking the cyclosporine." c. "If I develop acute rejection episode, I will need additional types of drugs." d. "The drugs are combined to inhibit different ways the kidney can be rejected."

B

The nurse teaches a patient with liver cancer about high-protein, high-calorie diet choices. Which snack choice by the patient indicates that the teaching has been effective? a. Lime sherbet b. Blueberry yogurt c. Fresh strawberries d. Cream cheese bagel

B

The occupational health nurse is caring for an employee who reporting bilateral eye pain after a cleaning solution splashed into the employee's eyes. Which action will the nurse take? a. Apply cool compresses. b. Flush the eyes with saline. c. Apply antiseptic ophthalmic ointment to the eyes. d. Cover the eyes with dry sterile patches and shields.

B

The registered nurse (RN) is caring for a patient who is living with HIV and admitted with tuberculosis. Which task can the RN delegate to unlicensed assistive personnel (UAP)? a. Teach the patient how to dispose of tissues with respiratory secretions. b. Stock the patient's room with the necessary personal protective equipment. c. Interview the patient to obtain the names of family members and close contacts. d. Tell the patient's family members the reason for the use of airborne precautions.

B

What action should the nurse complete before administering alendronate (Fosamax) to a patient with osteoporosis? a. Ask about any leg cramps or hot flashes. b. Assist the patient to sit up at the bedside. c. Be sure that the patient has recently eaten. d. Administer the ordered calcium carbonate.

B

What is the rationale for a nurse lifting, rather than pulling, a patient up in bed? a. Lifting a patient allows a brief period of increased capillary circulation. b. Lifting a patient prevents tissue injury from shearing forces. c. Pulling a patient up in bed decreases tissue ischemia and hypoxia. d. Pulling a patient up in bed promotes capillary blood flow.

B

What should the nurse assess to evaluate the effectiveness of treatment for the patient's myopia and presbyopia? a. Strength of the eye muscles. b. Both near and distant vision. c. Cloudiness in the eye lenses. d. Intraocular pressure changes.

B

What should the nurse teach a patient with repeated hordeolum about how to prevent further infection? a. Apply cold compresses. b. Discard all used eye cosmetics. c. Wash the eyebrows with an antiseborrheic shampoo. d. Be examined for sexually transmitted infections (STIs).

B

When performing a skin assessment, the nurse notes angiomas on the chest of an older patient. Which action should the nurse take next? a. Suggest an appointment with a dermatologist. b. Assess the patient for evidence of liver disease. c. Teach the patient about skin changes with aging. d. Discuss the use of sunscreen to prevent skin cancers.

B

When the patient turns his head quickly during the admission assessment, the nurse observes nystagmus. What is the indicated nursing action? a. Assess the patient with a Rinne test. b. Place a fall-risk bracelet on the patient. c. Ask the patient to watch the mouths of staff when they are speaking. d. Remind unlicensed assistive personnel to speak loudly to the patient.

B

Which action can the nurse working in the emergency department delegate to an experienced unlicensed assistive personnel (UAP)? a. Ask a patient with decreased visual acuity about medications taken at home. b. Perform Snellen testing of visual acuity for a patient with a history of cataracts. c. Obtain information from a patient about any history of childhood ear infections. d. Inspect a patient's external ear for redness, swelling, or presence of skin lesions.

B

Which action could the registered nurse (RN) who is working in the clinic delegate to a licensed practical/vocational nurse (LPN/VN)? a. Evaluate a patient's ability to administer eyedrops. b. Check a patient's visual acuity using a Snellen chart. c. Inspect a patient's external ear for signs of irritation caused by a hearing aid. d. Teach a patient with otosclerosis about use of sodium fluoride and vitamin D.

B

Which action for the care of a patient who has scleroderma can the registered nurse (RN) delegate to unlicensed assistive personnel (UAP)? a. Monitor for difficulty in breathing. b. Document the patient's oral intake. c. Check finger strength and movement. d. Apply capsaicin (Zostrix) cream to hands.

B

Which action should the nurse include in the plan of care for a patient with a new diagnosis of rheumatoid arthritis (RA)? a. Instruct the patient to purchase a soft mattress. b. Encourage the patient to take a nap in the afternoon. c. Teach the patient to use lukewarm water when bathing. d. Suggest exercise with light weights several times daily.

B

Which action should the nurse take first when a patient is seen in the outpatient clinic with neck pain? a. Provide information about therapeutic neck exercises. b. Ask about numbness or tingling of the hands and arms. c. Suggest the patient alternate the use of heat and cold to the neck. d. Teach about the use of nonsteroidal antiinflammatory drugs (NSAIDs).

B

Which action should the nurse take when repositioning the patient who has just had a laminectomy and discectomy? a. Instruct the patient to move the legs before turning the rest of the body. b. Place a pillow between the patient's legs and turn the entire body as a unit. c. Have the patient turn by grasping the side rails and pulling the shoulders over. d. Turn the patient's head and shoulders first, followed by the hips, legs, and feet.

B

Which action should the urgent care nurse take for a patient with a possible knee meniscus injury? a. Encourage bed rest for 24 to 48 hours. b. Apply an immobilizer to the affected leg. c. Avoid palpation or movement of the knee. d. Administer intravenous opioids for pain management.

B

Which action will the nurse take when performing ear irrigation for a patient with cerumen impaction? a. Assist the patient to a supine position for the irrigation. b. Fill the irrigation syringe with body-temperature solution. c. Use a sterile applicator to clean the ear canal before irrigating. d. Occlude the ear canal completely with the syringe while irrigating.

B

Which assessment finding alerts the nurse to provide patient teaching about cataract development? a. Unequal pupil size b. Sensitivity to light c. Loss of peripheral vision d. History of hyperthyroidism

B

Which assessment finding for a 55-yr-old patient should alert the nurse to the presence of osteoporosis? a. Bowed legs b. Loss of height c. Report of frequent falls d. Aversion to dairy products

B

Which assessment finding for a patient using naproxen (Naprosyn) to treat osteoarthritis is likely to require a change in medication? A. The patient has gained 3 lb B. The patient has dark-colored stools C. The patient's pain affects multiple joints D. The patient uses capsaicin cream (Zostrix)

B

Which assessment information should indicate to the nurse that a patient with an exacerbation of rheumatoid arthritis (RA) is experiencing a side effect of prednisone? a. The patient has joint pain and stiffness. b. The patient's blood glucose is 165 mg/dL. c. The patient has experienced a recent 5-pound weight loss. d. The patient's erythrocyte sedimentation rate (ESR) has increased.

B

Which equipment does the nurse need to perform a Rinne test? a. Otoscope b. Tuning fork c. Audiometer d. Ticking watch

B

Which example should the nurse use to explain an infant's "passive immunity" to a new mother? a. Vaccinations b. Breastfeeding c. Stem cells in peripheral blood d. Exposure to communicable diseases

B

Which information should the nurse include when preparing teaching materials for a patient who has an exacerbation of rheumatoid arthritis? a. Affected joints should not be exercised when pain is present b. Applying cold packs before exercise may decrease joint pain c. Exercises should be performed passively by someone other than the patient d. Walking may substitute for range-of-motion (ROM) exercises on some days

B

Which information should the nurse include when teaching a patient with newly diagnosed ankylosing spondylitis (AS) about managing the condition? a. Exercise by taking long walks. b. Do daily deep-breathing exercises. c. Sleep on the side with hips flexed. d. Take frequent naps during the day.

B

Which information should the nurse include when teaching a patient with newly diagnosed systemic exertion intolerance disease (SEID) about self-management? a. Symptoms usually progress as patients become older. b. A gradual increase in daily exercise may help decrease fatigue. c. Avoid use of over-the-counter antihistamines or decongestants. d. A low-residue, low-fiber diet will reduce any abdominal distention.

B

Which information will the nurse provide to the patient scheduled for refractometry? a. "You should not take any of your eye medicines before the examination." b. "You will need to wear sunglasses for a few hours after the examination." c. "The doctor will shine a bright light into your eye during the examination." d. "The surface of your eye will be numb while the doctor does the examination."

B

Which integumentary assessment data from an older patient admitted with bacterial pneumonia should be of concern to the nurse? a. Brown macules on extremities b. Reports a history of allergic rashes c. Skin wrinkled with tenting on both hands d. Longitudinal nail ridges and sparse scalp hair

B

Which nursing activity is appropriate for the registered nurse (RN) working in the eye clinic to delegate to experienced unlicensed assistive personnel (UAP)? a. Instilling antiviral drops for a patient with a corneal ulcer b. Application of a warm compress to a patient's hordeolum c. Instruction about hand washing for a patient with herpes keratitis d. Checking for eye irritation in a patient with possible conjunctivitis

B

Which of the following statements would be appropriate for a nurse to make to a patient about a lice infestation? a. "Only dirty people have lice." b. "Anyone can have lice." c. "Lice do not like to live on humans." d. "Lice are a form of fungus."

B

Which patient arriving at the urgent care center will the nurse assess first? a. Patient who is reporting that the left eyelid has just started to droop b. Patient with acute right eye pain that began while using power tools c. Patient with purulent left eye discharge and conjunctival inflammation d. Patient who has redness, crusting, and swelling along the lower right lid margin

B

Which patient should the nurse assess first? a. Patient with urticaria after receiving an IV antibiotic b. Patient who is sneezing after subcutaneous immunotherapy c. Patient who has graft-versus-host disease and severe diarrhea d. Patient with multiple chemical sensitivities with muscle stiffness

B

Which result for a patient with systemic lupus erythematosus (SLE) should the nurse identify as most important to communicate to the health care provider? a. Decreased C-reactive protein (CRP) b. Elevated blood urea nitrogen (BUN) c. Positive antinuclear antibodies (ANA) d. Positive lupus erythematosus cell prep

B

Which statement by a patient who has had an above-the-knee amputation indicates the nurse's discharge teaching has been effective? a. "I should elevate my residual limb on a pillow 2 or 3 times a day." b. "I should lie flat on my abdomen for 30 minutes 3 or 4 times a day." c. "I should change the limb sock when it becomes soiled or each week." d. "I should use lotion on the stump to prevent skin drying and cracking."

B

Which statement by a patient with bacterial conjunctivitis indicates a need for further teaching? a. "I will wash my hands often during the day." b. "I will remove my contact lenses at bedtime." c. "I will not share towels with my friends or family." d. "I will monitor my family for eye redness or drainage."

B

Which statement by a patient with systemic lupus erythematosus (SLE) indicates the patient understands the nurse's teaching about the condition? a. "I will exercise even if I am tired." b. "I will use sunscreen when I am outside." c. "I should avoid nonsteroidal antiinflammatory drugs." d. "I should take birth control pills to avoid getting pregnant."

B

Which statement by a patient would alert the nurse to a risk for decreased immune function? a. "I had a chest x-ray 6 months ago." b. "I had my spleen removed after a car accident." c. "I take one baby aspirin every day to prevent stroke." d. "I usually eat eggs or meat for at least two meals a day."

B

Which task can the nurse assign to unlicensed assistive personnel (UAP) who are working in the orthopedic clinic? a. Grade leg muscle strength for a patient with back pain. b. Obtain blood sample for uric acid from a patient with gout. c. Perform straight-leg-raise testing for a patient with sciatica. d. Check for knee joint crepitation before arthroscopic surgery.

B

Which topic will the nurse teach after a patient has had outpatient cataract surgery and lens implantation? a. Use of oral opioids for pain control b. Administration of corticosteroid drops c. Need for bed rest for 1 to 2 days after the surgery d. Importance of coughing and deep breathing exercises

B

A patient who is diagnosed with acquired immunodeficiency syndrome (AIDS) tells the nurse, "I feel obsessed with morbid thoughts about dying." Which response by the nurse is appropriate? a. "Thinking about dying will not improve the course of AIDS." b. "Do you think that taking an antidepressant might be helpful?" c. "Can you tell me more about the thoughts that you are having?" d. "It is important to focus on the good things about your life now."

C

A new clinic patient with joint swelling and pain is having diagnostic tests. Which test should the nurse identify as specific to systemic lupus erythematosus? a. Rheumatoid factor (RF) b. Antinuclear antibody (ANA) c. Anti-Smith antibody (Anti-Sm) d. Lupus erythematosus (LE) cell prep

C

A patient arrived at the emergency department after tripping over a rug and falling at home. Which finding should the nurse identify as most important to communicate to the health care provider? a. There is bruising at the shoulder area. b. The patient reports arm and shoulder pain. c. The right arm appears shorter than the left. d. There is decreased shoulder range of motion.

C

A patient diagnosed with external otitis is being discharged from the emergency department with an ear wick in place. Which statement by the patient indicates a need for further teaching? a. "I will apply the eardrops to the cotton wick in the ear canal." b. "I can use aspirin or acetaminophen (Tylenol) for pain relief." c. "I will clean the ear canal daily with a cotton-tipped applicator." d. "I can use warm compresses to the outside of the ear for comfort."

C

A patient has inadequate nutrition due to painful oral ulcers. Which nursing action will be most effective in improving oral intake? a. Offer the patient frequent small snacks between meals. b. Assist the patient to choose favorite foods from the menu. c. Apply prescribed anesthetic gel to oral lesions before meals. d. Teach the patient about the importance of nutritional intake.

C

A patient in the dermatology clinic is scheduled for removal of a 15-mm multicolored and irregular mole from the upper back. The nurse should prepare the patient for which type of biopsy? a. Shave biopsy b. Punch biopsy c. Incisional biopsy d. Excisional biopsy

C

A patient in the emergency department reports being struck in the right eye with a fist. Which finding is a priority for the nurse to communicate to the health care provider? a. The patient reports a right-sided headache. b. The sclera on the right eye has broken blood vessels. c. The patient reports "a curtain" over part of the visual field. d. The area around the right eye is bruised and tender to the touch.

C

A patient informed of a positive rapid screening test result for human immunodeficiency virus (HIV) is anxious and does not appear to hear what the nurse is saying. What action by the nurse is most important at this time? a. Inform the patient about the available treatments. b. Teach the patient how to manage a possible drug regimen. c. Remind the patient to return for retesting to verify the results. d. Ask the patient to identify those persons who had intimate contact.

C

A patient is being discharged 4 days after hip arthroplasty using the posterior approach. Which patient action requires intervention by the nurse? a. Using crutches with a swing-to gait b. Sitting upright on the edge of the bed c. Leaning over to pull on shoes and socks d. Bending over the sink while brushing teeth

C

A patient is receiving IV antibiotics at home to treat chronic osteomyelitis of the left femur. Which statement by the patient should indicate to the nurse the need for additional teaching related to health maintenance? a. "I'm frustrated with this endless treatment!" b. "I will take my oral temperature twice a day." c. "I think my left foot is starting to droop down." d. "I use crutches to avoid weight bearing on the left leg."

C

A patient on chemotherapy and radiation for head and neck cancer has a WBC count of 1.9 × 103/μL, hemoglobin of 10.8 g/dL, and a platelet count of 99 × 103/μL. Based on the CBC results, what is the most serious clinical finding? a. Cough, rhinitis, and sore throat b. Fatigue, nausea, and skin redness at site of radiation c. Temperature of 101.9° F, fatigue, and shortness of breath d. Skin redness at site of radiation, headache, and constipation

C

A patient who has ovarian cancer is crying and tells the nurse, "My husband rarely visits. He just doesn't care." The husband tells the nurse that he does not know what to say to his wife. Which problem is appropriate for the nurse to address in the plan of care? a. Anxiety b. Death anxiety c. Difficulty coping d. Lack of knowledge

C

A patient who has severe pain with terminal pancreatic cancer is being cared for at home by family members. Which finding by the nurse indicates that teaching about pain management has been effective? a. The patient uses the ordered opioid pain medication whenever the pain is greater than 5 (0 to 10 scale). b. The patient agrees to take the medications by the IV route to improve analgesic effectiveness. c. The patient takes opioids around the clock on a regular schedule and uses additional doses when breakthrough pain occurs. d. The patient states that nonopioid analgesics may be used if the maximal dose of the opioid is reached without adequate pain relief.

C

A patient who is scheduled for a breast biopsy asks the nurse the difference between a benign tumor and a malignant tumor. Which answer by the nurse is accurate? a. "Benign tumors do not cause damage to other tissues." b. "Benign tumors are likely to recur in the same location." c. "Malignant tumors may spread to other tissues or organs." d. "Malignant cells reproduce more rapidly than normal cells."

C

A patient who received a corneal transplant 2 weeks ago calls the ophthalmology clinic to report that his vision has not improved with the transplant. Which action should the nurse take? a. Suggest the patient arrange a ride to the clinic immediately. b. Ask about the presence of "floaters" in the patient's visual field. c. Remind the patient it may take months to restore vision after transplant. d. Teach the patient to continue using prescribed pupil-dilating medications.

C

A patient who slipped and fell in the shower at home has a proximal left humerus fracture immobilized with a sling. Which intervention should the nurse include in the plan of care? a. Use surgical net dressing to hang the arm from an IV pole. b. Immobilize the fingers of the left hand with gauze dressings. c. Assess the left axilla and change absorbent dressings as needed. d. Assist the patient in passive range of motion (ROM) for the right arm.

C

A patient who takes multiple medications develops acute gout arthritis. Which medication should the nurse discuss with the health care provider before administering a prescribed dose? a. sertraline (Zoloft). b. famotidine (Pepcid) c. hydrochlorothiazide. d. oxycodone (Roxicodone).

C

A patient who uses injectable illegal drugs asks the nurse how to prevent acquired immunodeficiency syndrome (AIDS). Which response by the nurse is most accurate? a. "Clean drug injection equipment before each use." b. "Ask those who share equipment to be tested for HIV." c. "Consider participating in a needle-exchange program." d. "Avoid sexual intercourse when using injectable drugs."

C

A patient with a complex pelvic fracture from a motor vehicle crash is on bed rest. Which assessment finding should indicate to the nurse a potential complication of the fracture? a. The patient states the pelvis feels unstable. b. The patient reports pelvic pain with palpation. c. Abdomen is distended, and bowel sounds are absent. d. Ecchymoses are visible across the abdomen and hips.

C

A patient with a head injury after a motorcycle crash arrives in the emergency department (ED) reporting shortness of breath and severe eye pain. Which action will the nurse take first? a. Assess cranial nerve functions. b. Administer the prescribed analgesic. c. Check the patient's oxygen saturation. d. Examine the eye for evidence of trauma.

C

A patient with a right retinal detachment had a pneumatic retinopexy procedure. Which information will the nurse include in the discharge teaching plan? a. The use of eye patches to reduce movement of the operative eye b. The need to wear dark glasses to protect the eyes from bright light c. The purpose of maintaining the head resting in a prescribed position d. The procedure for dressing changes when the eye dressing is saturated

C

A patient with an acute attack of gout in the right great toe has a new prescription for probenecid. Which information about the patient's home routine should the nurse understand indicates a need for teaching regarding gout management? a. The patient sleeps 8-10 hours each night. b. The patient usually eats beef once a week. c. The patient takes one aspirin a day to prevent angina. d. The patient usually drinks about 3 quarts water each day.

C

A patient with cancer is eating very little due to altered taste sensation. Which nursing action would address the cause of the patient problem? a. Add protein powder to foods such as casseroles. b. Tell the patient to eat foods that are high in nutrition. c. Avoid giving the patient foods that are strongly disliked. d. Add spices to enhance the flavor of foods that are served.

C

A patient with dark skin has been admitted to the hospital with acute decompensated heart failure. How would the nurse assess this patient for cyanosis? a. Assess the skin color of the earlobes. b. Apply pressure to the palms of the hands. c. Check the lips and oral mucous membranes. d. Examine capillary refill time of the nail beds.

C

A patient with hypertension and gout has a red, painful right great toe. Which action should the nurse include in the plan of care for this patient? a. Gently palpate the toe to assess swelling. b. Use pillows to keep the right foot elevated. c. Use a footboard to hold bedding away from the toe. d. Teach the patient to avoid acetaminophen (Tylenol).

C

A patient with metastatic colon cancer has severe vomiting after each administration of chemotherapy. Which action by the nurse is appropriate? a. Have the patient eat large meals when nausea is not present. b. Offer dry crackers and carbonated fluids during chemotherapy. c. Administer prescribed antiemetics 1 hour before the treatments. d. Give the patient a glass of a citrus fruit beverage during treatments.

C

A patient with rheumatoid arthritis (RA) tells the clinic nurse about having chronically dry eyes. Which action should the nurse take? a. Ask the HCP about discontinuing methotrexate. b. Remind the patient that RA is a chronic health condition. c. Suggest the patient use over-the-counter (OTC) artificial tears. d. Teach the patient about adverse effects of the RA medications.

C

A pedestrian who was hit by a car is admitted to the emergency department with possible right lower leg fractures. What initial action should the nurse take? a. Elevate the right leg. b. Splint the lower leg. c. Assess the pedal pulses. d. Verify tetanus immunization.

C

A teenaged male patient who is on a wrestling team is examined by the nurse in the clinic. Which assessment finding would prompt the nurse to teach the patient about the importance of not sharing headgear to prevent the spread of pediculosis? a. Ringlike rashes with red, scaly borders over the entire scalp b. Red, hivelike papules and plaques with circumscribed borders c. Papular, wheal-like lesions with white deposits on the hair shaft d. Patchy areas of alopecia with small vesicles and excoriated areas

C

After a motorcycle accident, a patient arrives in the emergency department with severe swelling of the left lower leg. Which action should the nurse take first? a. Elevate the leg on 2 pillows. b. Apply a compression bandage. c. Assess leg pulses and sensation. d. Place ice packs on the lower leg.

C

After change-of-shift report on the oncology unit, which patient should the nurse assess first? a. Patient who has a platelet count of 82,000/μL after chemotherapy. b. Patient who has xerostomia after receiving head and neck radiation. c. Patient who is neutropenic and has a temperature of 100.5° F (38.1° C). d. Patient who is worried about getting the prescribed long-acting opioid on time.

C

After completing the health history, how should the nurse begin to assess the musculoskeletal system? a. Feel for the presence of crepitus during joint movement. b. Have the patient move the extremities against resistance. c. Observe the patient's body build and muscle configuration. d. Check active and passive range of motion for the extremities.

C

After laminectomy with a spinal fusion to treat a herniated disc, a patient reports numbness and tingling of the right lower leg. What action should the nurse take? a. Elevate the right leg on two pillows. b. Obtain vital signs for indication of hemorrhage. c. Review the preoperative assessment data in the health record. d. Turn the patient to the left to relieve pressure on the right leg.

C

After the health care provider recommends amputation for a patient who has nonhealing ischemic foot ulcers, the patient tells the nurse that he would rather die than have an amputation. Which response by the nurse is best? a. "You are upset, but you may lose the foot anyway." b. "Many people are able to function with a foot prosthesis." c. "Tell me what you know about your options for treatment." d. "If you do not want an amputation, you do not have to have it."

C

An elderly patient has severe xerosis. What topic should the nurse include in a teaching plan for this patient? a. Take a hot bath every day. b. Use fabric softeners when laundering clothing. c. Apply skin lotions after a bath. d. Maintain a warm environment.

C

An older adult patient has a prescription for cyclosporine following a kidney transplant. Which information in the patient's health history has implications for planning patient teaching about the safe use of cyclosporine? a. The patient restricts salt to 2 grams per day. b. The patient eats green leafy vegetables daily. c. The patient drinks grapefruit juice every day. d. The patient drinks 3 to 4 quarts of fluid each day.

C

An older patient who is being admitted to the hospital repeatedly asks the nurse to "speak up so that I can hear you." Which action should the nurse take? a. Increase the speaking volume. b. Overenunciate while speaking. c. Speak normally but more slowly. d. Use more facial expressions when talking.

C

Diagnostic testing is recommended for skin lesions when? A. Percussion reveals an abnormal sound B. A health history cannot be obtained C. A more definitive diagnosis is needed D. Treatment with prescribed medication has failed

C

During the physical examination of a patient's skin, the nurse would a. use a flashlight in a poorly lit room. b. note cool, moist skin as a normal finding. c. pinch up a fold of skin to assess for turgor. d. perform a lesion-specific examination first and then a general inspection.

C

In caring for the patient after a spinal fusion, the nurse recognizes that interventions for this surgery differ from a simple laminectomy in that: A. Body alignment is maintained by the fusion procedure B. Earlier ambulation is permitted because the spine is more stabilized C. The donor site for the bone graft may be more painful than the spinal incision D. Teaching regarding body mechanics and prevention of future back injuries is not as critical

C

The charge nurse is assigning semiprivate rooms for new admissions. Which patient could safely be assigned as a roommate for a patient who has acute rejection of an organ transplant? a. A patient who has viral pneumonia b. A patient with second-degree burns c. A patient with an anaphylactic reaction d. A patient with graft-versus-host disease

C

The goals of cancer treatment are based on the principle that a. surgery is the single most effective treatment for cancer. b. initial treatment is always directed toward cure of the cancer. c. a combination of treatment modalities is effective for controlling many cancers. d. although cancer cure is rare, quality of life can be increased with treatment modalities.

C

The home health nurse notices irregular patterns of bruising at different stages of healing on an older patient's body. Which action should the nurse take first? a. Ensure the patient wears shoes with nonslip soles. b. Discourage using throw rugs throughout the house. c. Talk with the patient alone and ask about the bruising. d. Suggest that the health care provider prescribe radiographs.

C

The nurse assesses a patient who has just arrived in the postanesthesia recovery area (PACU) after a blepharoplasty. Which assessment data should be reported to the surgeon immediately? a. The patient reports incisional pain. b. The patient's heart rate is 100 beats/min. c. The skin around the incision is pale and cold. d. The patient is unable to sense touch on the eyelids.

C

The nurse at the outpatient surgery unit obtains the following information about a patient who is scheduled for cataract extraction and implantation of an intraocular lens. Which information is important to report to the health care provider before the procedure? a. The patient has had blurred vision for 3 years. b. The patient has not eaten anything for 8 hours. c. The patient takes antihypertensive medications. d. The patient gets nauseated with general anesthesia.

C

Which finding in a patient with a Colles' fracture of the left wrist should the nurse identify as most important to communicate immediately to the health care provider? a. The patient reports severe pain. b. Swelling is noted around the wrist. c. Capillary refill to the fingers is slow. d. The wrist has a deformed appearance.

C

The nurse instructs a patient who has osteosarcoma of the tibia about a scheduled above-the-knee amputation. Which patient statement indicates to the nurse that additional teaching is needed? a. "I will need to participate in physical therapy after surgery." b. "I wish I did not need to have chemotherapy after this surgery." c. "I did not have this bone cancer until my leg broke a week ago." d. "I can use the patient-controlled analgesia (PCA) to manage postoperative pain."

C

The nurse is caring for a patient diagnosed with stage I colon cancer. When assessing the need for psychologic support, which question by the nurse will provide the most information? a. "How long ago were you diagnosed with this cancer?" b. "Do you have any concerns about body image changes?" c. "Can you tell me what has been helpful when coping with past stressful events?" d. "Are you familiar with the stages of emotional adjustment to cancer of the colon?"

C

The nurse is caring for a patient who has a pelvic fracture and an external fixation device. How should the nurse perform assessment of pressure areas and provide skin care to the patient's back and sacrum? a. Ask the patient to turn to the side independently. b. Defer back assessment until the patient is ambulatory. c. Have the patient lift the back and buttocks using a trapeze. d. Roll the patient over to the side by pushing on the patient's hips.

C

The nurse is caring for a patient with left-sided lung cancer. Which finding would be most important for the nurse to report to the health care provider? a. Hematocrit of 32% b. Pain with deep inspiration c. Serum sodium of 126 mEq/L d. Decreased breath sounds on left side

C

The nurse learns that a newly admitted patient has functional blindness and that the spouse has cared for the patient for many years. What is the nurse's most important action during the initial assessment? a. Obtain more information about the cause of the patient's vision loss. b. Obtain information from the spouse about the patient's special needs. c. Make eye contact with the patient and ask about any need for assistance. d. Perform an evaluation of the patient's visual acuity using a Snellen chart.

C

The nurse palpates enlarged cervical lymph nodes on a patient diagnosed with acute human immunodeficiency virus (HIV) infection. Which action would be appropriate for the nurse to take? a. Instruct the patient to apply ice to the neck. b. Tell the patient a secondary infection is present. c. Explain to the patient that this is an expected finding. d. Request that an antibiotic be prescribed for the patient.

C

The nurse prepares to obtain a culture from a patient who has a possible fungal infection on the foot. Which items should the nurse gather for this procedure? a. Sterile gloves b. Patch test instruments c. Cotton-tipped applicators d. Syringe and intradermal needle

C

The nurse receives change-of-shift report on the oncology unit. Which patient should the nurse assess first? a. A 35-yr-old patient who has wet desquamation associated with abdominal radiation b. A 42-yr-old patient who is sobbing after receiving a new diagnosis of ovarian cancer c. A 24-yr-old patient who received neck radiation and has blood oozing from the neck d. A 56-yr-old patient who developed a new pericardial friction rub after chest radiation

C

The nurse reviewing a clinic patient's medical record notes that the patient missed the previous appointment for weekly immunotherapy. Which action by the nurse is appropriate? a. Schedule an additional dose the following week. b. Administer the scheduled dosage of the allergen. c. Consult with the health care provider about giving a lower allergen dose. d. Reevaluate the patient's sensitivity to the allergen with a repeat skin test.

C

The nurse should anticipate the need to teach a patient who has osteoarthritis (OA) about which medication? a. Prednisone b. Adalimumab (Humira) c. Capsaicin cream (Zostrix) d. Sulfasalazine (Azulfidine)

C

The nurse teaches a patient about application of corticosteroid cream to an area of contact dermatitis on the right leg. Which patient action indicates that further teaching is needed? a. The patient takes a tepid bath before applying the cream. b. The patient spreads the cream using a downward motion. c. The patient applies a thick layer of the cream to the affected skin. d. The patient covers the area with a dressing after applying the cream.

C

The nurse teaches a patient who is scheduled for a prostate needle biopsy about the procedure. Which statement by the patient indicates that teaching was effective? a. "The biopsy will remove the cancer in my prostate gland." b. "The biopsy will determine how much longer I have to live." c. "The biopsy will help decide the treatment for my enlarged prostate." d. "The biopsy will indicate whether the cancer has spread to other organs."

C

The nurse teaches a patient with osteoarthritis (OA) of the hip about how to manage the OA. Which patient statement indicates to the nurse a need for additional teaching? a. "A shower in the morning will help relieve stiffness." b. "I can exercise every day to help maintain joint mobility." c. "I will take 1 gram of acetaminophen (Tylenol) every 4 hours." d. "I can use a cane to decrease the pressure and pain in my hip."

C

The nurse teaches a postmenopausal patient with stage III breast cancer about the expected outcomes of cancer treatment. Which patient statement indicates that the teaching has been effective? a. "After cancer has not recurred for 5 years, it is considered cured." b. "The cancer will be cured if the entire tumor is surgically removed." c. "I will need follow-up examinations for many years after treatment before I can be considered cured." d. "Cancer is never cured, but the tumor can be controlled with surgery, chemotherapy, and radiation."

C

The reason newborns are protected for the first 3 months of life from bacterial infections is because of the maternal transmission of a. IgA. b. IgE. c. IgG. d. IgM.

C

The second day after admission with a fractured pelvis, a patient suddenly develops confusion. Which action should the nurse take first? a. Take the blood pressure. b. Check the O2 saturation. c. Assess patient orientation. d. Observe for facial asymmetry.

C

There is one opening in the schedule at the dermatology clinic, and four patients are seeking appointments today. Which patient will the nurse schedule for the available opening? a. 42-yr-old with itching after using topical fluorouracil on the nose b. 50-yr-old with skin redness after having a chemical peel 3 days ago c. 38-year old with a 7-mm nevus on the face that has recently become darker d. 62-yr-old with multiple small, soft, pedunculated papules in both axillary areas

C

Trends in the incidence and death rates of cancer include the fact that a. a higher percent of women than men have lung cancer. b. lung cancer is the most common type of cancer in men. c. blacks have a higher death rate from cancer than whites. d. breast cancer is the leading cause of cancer deaths in women.

C

What is the most effective method to prevent the spread of infection to others when the nurse is changing the dressing over a wound infected with Staphylococcus aureus? a. Change the dressing using sterile gloves. b. Apply antibiotic ointment over the wound. c. Wash hands and properly dispose of soiled dressings. d. Soak the dressing in sterile normal saline before removal.

C

What is the safest technique for the nurse to use when assisting a blind patient to ambulate to the bathroom? a. Lead the patient slowly to the bathroom, holding on to the patient by the arm. b. Stay beside the patient and describe any obstacles on the path to the bathroom. c. Walk slightly ahead of the patient, allowing the patient to hold the nurse's elbow. d. Have the patient place a hand on the nurse's shoulder and guide the patient forward.

C

What should the nurse include in the teaching plan for ae patient who has acute low back pain and muscle spasms? a. Keep both feet flat on the floor when prolonged standing is required. b. Twist gently from side to side to maintain range of motion in the spine. c. Keep the head elevated slightly and flex the knees when resting in bed. d. Avoid the use of cold packs because they will exacerbate the muscle spasms.

C

What should the nurse include when teaching a patient who has undergone a left tympanoplasty? a. "Remain on bed rest." b. "Keep your head elevated." c. "Avoid blowing your nose." d. "Irrigate your left ear canal."

C

What should the nurse teach a patient with recurrent staphylococcal and seborrheic blepharitis to do? a. Irrigate the eyes with saline solution. b. Schedule an appointment for eye surgery. c. Use a gentle baby shampoo to clean the eyelids. d. Apply cool compresses to the eyes three times daily.

C

What suggestion should the nurse make to a group of women with rheumatoid arthritis (RA) about managing activities of daily living? a. Protect the knee joints by sleeping with a small pillow under the knees. b. Strengthen small hand muscles by wringing out sponges or washcloths. c. Avoid activities requiring repetitive use of the same muscles and joints. d. Stand rather than sit when performing daily household and yard chores.

C

When a patient arrives in the emergency department with a facial fracture, which action should the nurse take first? a. Assess for nasal bleeding and pain. b. Apply ice to the face to reduce swelling. c. Use a cervical collar to stabilize the spine. d. Check the patient's alertness and orientation.

C

When assessing a new patient at the outpatient clinic, the nurse notes dry, scaly skin; thin hair; and thick, brittle nails. What is the nurse's most important action? a. Instruct the patient about the importance of nutrition for skin health. b. Make a referral to a podiatrist so that the nails can be safely trimmed. c. Consult with the health care provider about the need for further diagnostic testing. d. Teach the patient about using moisturizing creams and lotions to decrease dry skin.

C

When assessing the nutritional-metabolic pattern in relation to the skin, the nurse asks the patient about a. joint pain. b. the use of moisturizing shampoo. c. recent changes in wound healing. d. self-care habits related to daily hygiene.

C

When the nurse brings medications to a patient with rheumatoid arthritis, the patient refuses the prescribed methotrexate. The patient tells the nurse, "My arthritis isn't that bad yet. The side effects of methotrexate are worse than the arthritis." What is the most appropriate response by the nurse? a. "You have the right to refuse to take the methotrexate." b. "Methotrexate is less expensive than some of the newer drugs." c. "It is important to start methotrexate early to decrease the extent of joint damage." d. "Methotrexate is effective and has fewer side effects than some of the other drugs."

C

Which action included in the care of a patient after laminectomy can the nurse delegate to experienced unlicensed assistive personnel (UAP)? a. Check ability to plantar and dorsiflex the foot. b. Determine the patient's readiness to ambulate. c. Log roll the patient from side to side every 2 hours. d. Ask about pain management with the patient-controlled analgesia (PCA).

C

Which action should the nurse take when teaching a patient with mild presbycusis? a. Use patient education handouts rather than discussion. b. Use a high-pitched tone of voice to provide instructions. c. Ask for permission to turn off the television before teaching. d. Wait until family members have left before initiating teaching.

C

Which assessment finding should the nurse report to the health care provider? a. Visible cone of light b. Dry skin in the ear canal c. A blue-tinged tympanum d. Cerumen in the auditory canal

C

Which discharge instruction should the emergency department nurse include for a patient with a sprained ankle? a. Keep the ankle loosely wrapped with gauze. b. Apply a heating pad to reduce muscle spasms. c. Use pillows to elevate the ankle above the heart. d. Gently move the ankle through the range of motion.

C

Which exposure by the nurse is most likely to require postexposure prophylaxis when the patient's human immunodeficiency virus (HIV) status is unknown? a. Bite to the arm that does not result in open skin b. Splash into the eyes while emptying a bedpan containing stool c. Needle stick with a needle and syringe used for a venipuncture d. Contamination of open skin lesions with patient vaginal secretions

C

Which information about intradermal skin testing should the nurse teach to a patient with possible allergies? a. "Do not eat anything for about 6 hours before the testing." b. "Take an oral antihistamine about an hour before the testing." c. "Plan to wait in the clinic for 20 to 30 minutes after the testing." d. "Reaction to the testing will take about 48 to 72 hours to occur."

C

Which information from a patient's health history should the nurse identify as a risk factor for septic arthritis? a. Recently visited South America b. Several knee injuries as a teenager c. Sexually active with several partners D. Has a parent who has rheumatoid arthritis

C

Which information obtained during the nurse's assessment may indicate a patient's increased risk for musculoskeletal problems? a. The patient takes a multivitamin daily. b. The patient dislikes fruits and vegetables. c. The patient is 5 ft, 2 in tall and weighs 180 lb. d. The patient prefers whole milk to nonfat milk.

C

Which information should the nurse include in discharge instructions for a patient with comminuted left forearm fractures and a long-arm cast? a. Keep the left shoulder elevated on a pillow or cushion. b. Avoid nonsteroidal antiinflammatory drugs (NSAIDs). c. Call the health care provider for numbness of the hand. d. Keep the hand immobile to prevent soft tissue swelling.

C

Which information should the nurse include in the teaching plan for a patient diagnosed with basal cell carcinoma (BCC)? a. Treatment plans include watchful waiting. b. Screening for metastasis will be important. c. Minimizing sun exposure reduces risk for future BCC. d. Low-dose systemic chemotherapy is used to treat BCC.

C

Which patient statement indicates understanding of the nurse's teaching about a new short-arm synthetic cast? a. "I can remove the cast in 4 weeks using industrial scissors." b. "I should avoid moving my fingers until the cast is removed." c. "I will apply an ice pack to the cast over the fracture site off and on for 24 hours." d. "I can use a cotton-tipped applicator to rub lotion on any dry areas under the cast."

C

A factory line worker has repetitive strain syndrome in the left elbow. What topic should the nurse plan to include in patient teaching? a. Surgical options b. Elbow injections c. Wearing a left wrist splint d. Modifying arm movements

D

In reviewing a patient's medical record, the nurse notes that the last eye examination revealed an intraocular pressure of 28 mm Hg. What should the nurse plan to assess? a. Visual acuity b. Pupil reaction c. Color perception d. Peripheral vision

D

The charge nurse is observing a new nurse who is caring for a patient with vestibular disease. For what action by the nurse should the charge nurse intervene immediately? a. Facing the patient directly when speaking b. Speaking slowly and distinctly to the patient c. Administering both the Rinne and Weber tests d. Encouraging the patient to ambulate independently

D

The most effective method of administering a chemotherapy agent that is a vesicant is to a. give it orally. b. give it intraarterially. c. use an Ommaya reservoir. d. use a central venous access device.

D

The nurse assesses a 78-yr-old who uses naproxen (Aleve) daily for hand and knee osteoarthritis management. Which information should the nurse discuss with the health care provider for an urgent change in the treatment plan? a. Knee crepitation is noted with normal knee range of motion. b. Patient reports embarrassment about having Heberden's nodes. c. Patient's knee pain while golfing has increased over the last year. d. Laboratory results indicate blood urea nitrogen (BUN) is elevated.

D

The nurse at the eye clinic made a follow-up telephone call to a patient who underwent cataract extraction and intraocular lens implantation the previous day. Which information is the priority to communicate to the health care provider? a. The patient reports that the vision has not improved. b. The patient requests a prescription refill for next week. c. The patient feels uncomfortable wearing an eye patch. d. The patient reports eye pain rated 5 (on a 0 to 10 scale).

D

The nurse finds that a patient can flex the arms when no resistance is applied but is unable to flex against light resistance. How should the nurse document the patient's muscle strength level? a. 0 b. 1 c. 2 d. 3

D

The nurse is caring for a patient who is living with human immunodeficiency virus (HIV) and taking antiretroviral therapy (ART). Which information is most important for the nurse to address when planning care? a. The patient reports feeling "constantly tired." b. The patient reports having no side effects from the medications. c. The patient is unable to explain the effects of atorvastatin (Lipitor). d. The patient reports missing doses of tenofovir AF/emtricitabine (Descovy).

D

The nurse is interviewing a patient with contact dermatitis. Which finding indicates a need for patient teaching? a. The patient applies corticosteroid cream to pruritic areas. b. The patient adds oilated oatmeal to the bath water every day. c. The patient takes diphenhydramine at night for persistent itching. d. The patient uses bacitracin-neomycin-polymyxin on minor abrasions.

D

The nurse is working in an urgent care clinic that has standardized treatment protocols for implementation by nursing staff. After reviewing the history, physical assessment, and vital signs for a 60-yr-old patient as shown in the accompanying figure, which action should the nurse take first? a. Check the patient's blood glucose level. b. Take the blood pressure on the left arm. c. Use an irrigating syringe to clean the ear canals. d. Report a vision change to the health care provider.

D

The nurse notes darker skin pigmentation in the skinfolds of a middle-aged patient who has a body mass index of 40 kg/m2. What is the nurse's appropriate action? a. Discuss the use of drying agents to minimize infection risk. b. Instruct the patient about the use of mild soap to clean skinfolds. c. Teach the patient about treating fungal infections in the skinfolds. d. Ask the patient about a personal or family history of type 2 diabetes.

D

The nurse notes that a 59-yr-old female patient has lost 1 inch in height over the past 2 years. What diagnostic test should the nurse plan to discuss with the patient? a. Discography studies b. Myelographic testing c. Magnetic resonance imaging (MRI) d. Dual-energy x-ray absorptiometry (DXA)

D

The nurse obtains information about a hospitalized patient who is receiving chemotherapy for colorectal cancer. Which information about the patient alerts the nurse to discuss a possible change in cancer therapy with the health care provider? a. Frequent loose stools b. Nausea and vomiting c. Elevated white blood count (WBC) d. Increased carcinoembryonic antigen (CEA)

D

The nurse prepares to administer the following medications to a hospitalized patient with human immunodeficiency (HIV). Which medication is most important to administer at the scheduled time? a. Nystatin tablet b. Oral acyclovir (Zovirax) c. Aerosolized pentamidine (NebuPent) d. Oral tenofovir AF/emtricitabine/bictegravir (Biktarvy)

D

The nurse should assess the patient undergoing plasmapheresis for which clinical manifestation? a. Shortness of breath b. High blood pressure c. Transfusion reaction d. Extremity numbness

D

The nurse tells a friend who asks him to administer his allergy shots that a. it is illegal for nurses to administer injections outside of a medical setting. b. he is qualified to do it if the friend has epinephrine in an injectable syringe provided with his extract. c. avoiding the allergens is a more effective way of controlling allergies, and allergy shots are not usually effective. d. immunotherapy should only be administered in a setting where emergency equipment and drugs are available.

D

What accurately describes rejection after transplantation? a. Hyperacute rejection can be treated with OKT3. b. Acute rejection can be treated with sirolimus or tacrolimus. c. Chronic rejection can be treated with tacrolimus or cyclosporine. d. Hyperacute reaction can be avoided if crossmatching is done before transplantation.

D

What instructions about plasmapheresis should the nurse include in the teaching plan for a patient diagnosed with systemic lupus erythematosus (SLE)? a. Plasmapheresis counteracts recovery of IgG production. b. Plasmapheresis removes eosinophils and basophils from the blood. c. Plasmapheresis decreases the damage to organs from T lymphocytes. d. Plasmapheresis prevents inflammatory mediators from injuring tissues.

D

What question should the nurse include in a health history of a patient with a linear pattern of painful vesicles over the left thorax? a. "Do you remember being sunburned as a child?" b. "Are you a regular patron of tanning booths?" c. "Have you ever been diagnosed with acne?" d. "Did you have chickenpox when you were young?"

D

What should the nurse assess to evaluate the effectiveness of alendronate (Fosamax) therapy for a patient with Paget's disease? a. Oral intake b. Daily weight c. Grip strength d. Pain intensity

D

When grading muscle strength, the nurse records a score of 3/5, which indicates a. no detection of muscular contraction. b. a barely detectable flicker of contraction. c. active movement against full resistance without fatigue. d. active movement against gravity but not against resistance.

D

Which action should the nurse include in the plan of care for a patient who had a cemented right total knee arthroplasty? a. Avoid extension of the right knee beyond 120 degrees. b. Use a compression bandage to keep the right knee flexed. c. Teach about the need to avoid weight bearing for 4 weeks. d. Start progressive knee exercises to obtain 90-degree flexion.

D

Which menu choice by a patient with osteoporosis indicates the nurse's teaching about appropriate diet has been effective? a. Pancakes with syrup and bacon b. Whole wheat toast and fresh fruit c. Egg-white omelet and a half grapefruit d. Oatmeal with skim milk and fruit yogurt

D


Kaugnay na mga set ng pag-aaral

periods of nursing history - Monique

View Set

Span 102 Lección 14.2 ¿Cierto o falso?

View Set

Solving with the Distributive Property Assignment

View Set

1. A pénzügyi szektor alapvetései

View Set